• Shuffle
    Toggle On
    Toggle Off
  • Alphabetize
    Toggle On
    Toggle Off
  • Front First
    Toggle On
    Toggle Off
  • Both Sides
    Toggle On
    Toggle Off
  • Read
    Toggle On
    Toggle Off
Reading...
Front

Card Range To Study

through

image

Play button

image

Play button

image

Progress

1/228

Click to flip

Use LEFT and RIGHT arrow keys to navigate between flashcards;

Use UP and DOWN arrow keys to flip the card;

H to show hint;

A reads text to speech;

228 Cards in this Set

  • Front
  • Back
A 56 yo woman is found apneic and pulseless. Cardiac monitoring shows sinus tachycardia at a rate of 130 bpm. What actions are the appropriate next step in the management of this patient?

a. initiation of CPR and administration of a 500 ml fluid bolus of normal saline
b. intubation and the administration of amiodarone
c. initiation of CPR and the administration of atropine
d. Initiation of CPR and unsynchronized countershock with 200 joules
e. Initiation of CPR and synchronized countershock with 50 joules
a. Initiation of CPR and administration of a 500 ml fluid bolus of normal saline

Pt is in PEA and the most common cause of PEA is hypovolemia
Which one of the following is the most common cause of airway obstruction?
a. tracheal-laryngeal fracture
b. Foreign body
c. Pooled secretions
d. Prolapse of the tongue
e. Fractured mandible
d. Prolapse of the tongue
A 27 yo man with severe hypovolemia requires rapid fluid resuscitation. Which one of the following catheters would be ideal for this resuscitation effort?
a. 12 inch 8 French introducer
b. 5 1/2 inch 9 french introducer
c. 2 inch 14 gauge catheter
d. 12 inch 16 gauge catheter
e. 5 inch 16 gauge catheter
b. 5 1/2 inch 9 French introducer

the most rapid rate of fluid infusion occurs with the catheter of the widest diameter and shortest length
Rapid sequence induction (RSI) using lidocaine, thiopental and succinylcholine is most appropriate for which one of the following pts?
a. A 67 yo woman with sudden onset hemiplegia who is nonverbal with pooled secretions in her oropharynx and who appears slighly cyanotic, with a BP of 220/115 mmHg and a pulse of 60 bpm
b. a 24 yo man who was thrown 25 feet from a motorcycle and who was unconscious in the field and is still unconscious, with a BP of 86/46 and a pulse of 110 bpm
c. a 24 yo woman with severe exacerbation of her asthma and a BP of 170/100
d. a 92 yo man with sudden onset aphasia, pooled secretions, a BP of 90/60 and a pulse of 110 bpm
e. A 64 yo man with rapid onset tongue edema suspected to be caused by angioedema induced by therapy with a new ACE inhibitor
a. 67 yo woman with sudden onset hemiplegia who is nonverbal with pooled secretions in her oropharynx and who appears slightly cyanotic with a BP of 220/115 and a pulse of 60

Thiopental and Lidocaine are used in RSI when increased ICP are considered

Thiopental is contraindicated in pts with low BP
A 28 yo woman who is 38 weeks pregnant is resuscitated by paramedics and brought to the ED. The pt is hypotensive and groans spontaneously. What is the 1st step in treating this pt?
a. Administering a 500 ml fluid challenge
b. Performing a perimortem cesarean section
c. positioning the pt on her left side
d. administering 1 mg epinepherine IV
e. initiating rapid sequence induction (RSI)
c. Positioning pt on her left side

most common cause of hypotension in late pregnancy is compression of the IVC by the gravid uterus which results in decreased venous return and reduced cardiac output
A 50 yo man presents to the ED complaining of "palpitations." In the triage area, he collapses after complaining of shortness of breath and substrenal chest pain. Quick-look with a cardiac monitor reveals a regular, narrow QRS complex and a heart rate of 200 bpm. A tentative diagnosis of paroxysmal supraventricular tachycardia is made. What is the most appropriate 1st intervention?
a. Vagal maneuvers
b. Adenosine, 6 mg administered IV
c. Synchronized counterschock at 200 joules
d. Synchronized countershock at 50 joules
e. unsynchronized countershock at 200 Joules
d. Synchronized countershock at 50 joules

unstable paroxysmal SVT is treated with synchronized cardioversion starting at 50 joules
Which one of the following induction agents causes a dissociative state and is associated with relaxation of bronchial smooth muscle?
a. fentanyl
b. ketamine
c. Etomidate
d. Midazolam
e. Thiopental
b. ketamine

ketamine is a PCP derivative
Which one of the following means of providing supplemental oxygen allows the most precise adjustment of oxygen concentration?
a. bag-valve-mask system
b. nonrebreather mask
c. standard (simple) face mask
d. Venturi mask
e. Nasal Cannula
d. Venturi mask

the Venturi mask allows adjustment of inspired oxygen via adjustment of the oxygen flow rate
An elderly patient presents to the ED with severe pulmonary edema and an initial room-air oxygen saturation of 86%. Oxygen saturation increases to 98% with supplemental oxygen and bilevel positive aireay pressure (BiPAP) via face mask; however, the pt is clearly too fatigued to continue spontaneous respirations much longer and intubation is required. When commencing RSI, which one of the following actions is most appropriate?
a. The pt should be preoxygenated for several minutes with vigorous bag-valve-mask assisted ventilation
b. The pt should be preoxygenated for several minutes with a tight-fitting nonrebreathing mask, unassisted bag-valve-mask ventilation, or by several full volume breaths via a bag-valve-mask just before intubation
c. A nasogastric tube should be placed to remove the gastric contents prior to intubation (to decrease the risk of aspiration)
d. the sellick maneuver should be performed once the decision to intubate has been made
e. the pt should be aggressively ventilated throughout the RSI protocol
b. The pt should be preoxygenated for several minutes with a tight-fitting nonrebreathing mask, unassisted bag-valve-mask ventilation or by several full volume breaths via a bag-valve-mask just before intubation
A 26 yo man is brought to the ED because he felt dizzy, weak and faint while playing soccer. The pt says he has not experienced chest pain or shortness of breath, and denies antecedent drug use. His medical history is unremarkable. Review of systems reveals a recent decrease in exercise tolerance due to dyspnea. Which one of the following statements regarding this case is true?
a. The cause of this patient's near-syncope is most likely benign and he should be advised to follow up with his family doctor within 1 week
b. a urine drug screen will most likely reveal recent cocaine use, which could cause a toxic cardiomyopathy and precipitate the near-syncopal episode
c. A murmur may be present on physical exam and can be expected to increase in intensity with squatting
d. EKG findings of LVH along with physical exam findings of cardiac enlargement in this pt would suggest the diagnosis of athlete's heart syndrome
e. Despite an unremarkable physical exam, this pt should undergo an emergent echo and may require admission to the hospital for further evaluation
e. Despite an unremarkable physical exam, this pt should undergo an emergent echo and may require admission to the hospital for further evaluation

Most likely Dx = Hypertrophic cardiomyopathy which can cause syncope or near-syncope
Which of the following symptoms is present in mild to moderate mitral stenosis?
a. Fatigue
b. Exertional dyspnea
c. Chest pain
d. Peripheral edema
e. Dysphagia
B. Exertional dyspnea

A common complaint early in the course of mitral stensis
Which one of the following measures is most appropriate for the treatment of cardiogenic shock due to acute MI?
a. Oxygen, Nitrates, and Heparin
b. Dopamine and IV nitroglycerin
c. Emergent percutaneous transluminal coronary angioplasty (PTCA) following hemodynamic stabilization
d. Streptokinase, heparin, and IV fluids
c. Emergent percutaneous transluminal coronary angioplasty (PTCA) following hemodynamic stabilization

Cardiogenic shock is often a complication of transmural, anterior wall MI
Which one of the following signs or symptoms is characteristic of mitral regurgitation?
a. Head bobbing
b. A palpable systolic thrill and a ventricular heave
c. JVD with cannon a waves
d. Ascites
e. Diminished carotid upstroke
b. a palpable systolic thrill and a ventricular heave

Mitral regurg results in dilation and hypertrophy of the left ventricle which can be felt by palpation as a thrill and a heave
Which one of the following statements regarding syncope is true?
a. The dx is confirmed electrocardiographically in 50% of pts
b. Pts usually report a prodrome of palpitations or chest pain when the cause of the syncope is a conduction disturbance
c. pt may give a hx of recent medication change
d. pts may have subtle focla findings on neurologic examination
e. Admission to a monitored setting is required
c. pts may give a hx of recent medication change.
Which one of the following signs or symptoms is characteristic of aortic regurgitation?
a. Florid congestion of the cheeks
b. a widened pulse pressure
c. Kussmaul's sign
d. a 3rd heart sound (S3)
b. a widened pulse pressure

a widened pulse pressure is noted as the forceful ejection of blood in systole rapidly falls off d/t regurgitation thru the aortic valve in diastole
Which one of the following statements regarding thoracic aortic dissection is true?
a. The pt will most likely be hypotensive on initial presentation
b. ST segement elevation in teh anterior leads is a typical EKG finding
c. Pts are likely to describe chest pain that is ripping, is severe, worsens with inspiration and radiates to the right shoulder
d. Radiographic findings include a widened mediastinum with possible obliteration of the aoritc knob
e. Nitroprusside and dopamine are the therapeutic agents of choice
d. Radiographic findings include a widened mediastinum with possible obliteration of the aoritc knob

will also see displacement of the trachea to the right and a double density appearance of the aorta
A 58 yo woman presents to the ED after falling on an icy sidewalk. Her left ankle is painful and swollen. The pt denies any medical problems, stating that she is "never sick". Her BP is 165/108 and her heart rate is 80 bpm. The most reasonable approach for the ED physician to take would be to treat the patient's ankle injury and
a. administer nifedipine (10mg) to lower her BP and discharger her from the ED after her BP has decreased by 20% with the recommendation that she can be reevaluated by her family physician in 2 days
b. initiate IV antihypertensive medication
c. refer the pt to a nephrologist for evaluation of probably secondary HTN
d. discharge her from the ED with an antihypertensive medication
e. advise her to follow up with her family doctor
e. Advise her to follow up with her family doctor

pt may have undiagnosed essential HTN or her BP may be elevated as a result of pain and anxiety
Which one of the following statements regarding mitral stenosis is true?
a. It is sometimes heralded by the development of hemoptysis, which is often fatal
b. Pts have an enlarged point of maximal impulse d/t LVH
c. lifelong anticoagulation therapy is usually necessary
d. it develops in 40% of pts following rheumatic heart disease
e. It causes atrial enlargement and the development of a. fib which is usually well tolerated
d. it develops in 40% of pts following rheumatic heart disease
Which one of the following statements regarding Dressler's syndrome is true?
a. It occurs within 1-4 days of acute MI
b. It is commonly associated with a bloody pericardial effusion
c. It occurs from 1 week to several months following acute MI
d. It is a life threatening emergency and should be treated with emergency pericardiocentesis
e. It results in an increased risk for thoracic aortic dissection
c. It occurs from 1 week to several months following acute MI
A 30 yo woman with a history of asthma presents to the ED complaining of shortness of breath. She is afebrile and has diffuse wheezing on pulmonary exam. Although she is mildly tachypneic, she appears well, is not cyanotic, and is not in respiratory distress. On arrival, her peak expiratory flow rate is 250 L/min.
What is the best initial therapy for this pt?
a. nebulized albuterol
b. subcutaneous epi
c. nebulized albuterol and epi
d. nebulized albuterol and theophylline

The pt improves slightly after 2 treatments. She still has mild wheezing, feels mildly dyspneic, and has a PEFR of 280 L/min. What is the agent of choice in this situation?
a. nebulized albuterol
b. nebulized albuterol and oral prednisone
c. IV methylprednisolone
d. Subcutaneous epi
a. nebulized albuterol = 1st line treatment for asthma exacerbations

B. nebulized albuterol and oral prednisone
A pt in respiratory failure has just been intubated. The ventilator should be set to deliver a tidal volume of
a. 1-2 ml/kg
b. 5-10 ml/kg
c. 10-12 ml/kg
d. 20-25 ml/kg
c. 10-12 ml/kg

general rule is a tidal volume of 10-12 ml/kg to provide adequate ventilation for intubated pts
A pt presents to the ED with massive hemoptysis. He has a history of cancer of the left lung. Management of this patient might include which one of the following measures?
a. Positioning the pt with his right side down
b. Obtaining IV access pending hematocrit results
c. Selectively intubating the right mainstem bronchus
d. none of the above
c. Selectively intubating the right mainstem bronchus

Main goal of therapy for a pt with massive hemoptysis is to stabilize the pts respiratory and hemodynamic status while preventing the spread of blood throughout the lungs. If pt needs to be intubated, selectively intubate the right mainstem bronchus b/c this will allow ventilation of the unaffected lung
When should a chest radiograph be ordered for a patient with asthma?
a. When teh pt prevents with diffuse wheezing
b. When the pt has a history of cough
c. When the initial exam findings include tachypnea
d. When pulmonary exam reveals decreased breath sounds and egophony at the left lung base
d. When pulmonary exam reveals decreased breath sounds and egophony at the left lung base

consider xray if pt has: hypoxia, fever, symptoms that do not respond to bronchodilation, or a focal lung exam
A pt with COPD presents to the ED with mild respiratory distress. Which set of arterial blood gas findings is suggestive of respiratory failure in this patient?
a. pH = 7.41, pO2 = 70, pCO2 = 52
b. pH = 7.40, pO2 = 70, pCO2 = 52
c. pH = 7.28, pO2 = 70, pCO2 = 60
d. pH = 7.40, pO2 = 75, pCO2 = 60
c. pH = 7.28, pO2 = 70, pCO2 = 60

an increased pCO2 accompanied by a decreased pH is characteristic of respiratory failure in a pt with COPD
Which one of the following is consistent with a diagnosis of noncardiogenic pulmonary edema (NCPE)?
a. Peripheral edeam
b. Patchy alveolar infiltrates on chest radiograph
c. Jugular venous distention
d. A normal A-a gradient
b. Patchy alveolar infiltrates on chest radiograph

this is a bilateral finding with a normal heart size
Which one of the following medications is contraindicated in the treatment of a 50 yo asthmatic patient with a history of ischemic heart disease?
a. Ipratropium bromide
b. Ketamine
c. Albuterol
d. Heliox
b. Ketamine
Which one of the following symptoms is most suggestive of hypercapnia?
a. Agitation
b. Somnolence
c. Dyspnea
d. disorientation
b. somnolence

somnolence and lethergy are commonly seen in hypercapnic pts

agitation, dyspnea, disorientation and confusion are seen in hypoxic pts
Which one of the following statements regarding tension pneumothorax is true?
a. the pt may be asymptomatic
b. Tracheal deviation is an early and unmistakable sign
c. A chest radiograph should be ordered prior to initiating treatment
d. The pt may present with Pulseless electrical activity
d. The pt may present with Pulseless electrical activity

pts with a tension pneumo are critically ill, in severe respiratory distress and may be unconscious
The evaluation of a patient with COPD who is experiencing an acute exacerbation but does not demonstrate signs of respiraotry failure should always include:
a. PFT and Pulse ox
b. PFT and ABG
c. PFT, pulse ox and chest radiographs
d. PFT, ABG and Chest Radiographs
a. PFT and Pulse ox

PFT to evaluate peak expriatory flow and pulse ox to evaluate the degree of hypoxia and obstruction
Which one of the following statements regarding the treatment of patients with noncardiogenic pulmonary edema (NCPE) is true?
a. inspired oxygen concentration (FIO2) should be at least 90%
b. Positive end-expiratory pressure (PEEP) may improve oxygenation
c. Corticosteroids should be administered routinely and immediately
d. PEEP should NOT be used b/c it may result in pneumothroax
b. Positive end-expiratory pressure (PEEP) may improve oxygenation

too high of PEEP may result in barotrauma
A 50 yo previously healthy pt presents to the ED b/c he is experiencing mild, left sided Chest pain. The pt says that he has been having this pain for the past 3 months. A chest xray demonstrates a left pleural effusion of moderate size. A lateral decubitus view demonstrates that the effusion is free flowing and the pt is not in respiratory distress. The best initial management of this pt will include:
a. chest tube placement
b. thoracentesis
c. the administration of 100% oxygen and overnight observation
d. Gastrografin swallow
b. thoracentesis

procedure of choice for diagnosing and draining pleural effusions
a 72 yo pt is brought to the ED from a nursing home and is diagnosed with pneumonia. When selecting an antibiotic regimen, it is important to provide adequate coverage for which one of the following organisms?
a. Legionella
b. Mycoplasma
c. Pseudomonas
d. Coccidioides
c. Pseudomonas
a 47 yo man comes to the ED complaining of a cough and fever. The nurse informs the emergency physician that the pt is homeless and an IV drug user. He has a temp of 101 F and all of his other vitals are normal. Which one of the following actions should the emergency physician take 1st?
a. she should perform a thorough history and physical
b. She should send the pt to radiology for CXR
c. she should contact the ED social worker
d. She should put on a respiratory mask prior to entering the pts room
d. She should put on a respiratory mask prior to entering the pts room

b/c pt is complaining of cough and fever the dx might be pneumonia. Also should consider TB b/c he is homeless and an IV drug user
What is the most common cause of upper GI bleeding?
a. Mallory-Weiss tear
b. Esophageal varices
c. Peptic ulcer
d. Arteriovenous malfaormation
e. Aortoenteric fistula
c. Peptic ulcer
A 2 yo boy presents to the ED after ingestion of a calculator battery. A chest radiograph demonstrates that the battery is in the esophagus. What would be the most appropriate next course of action?
a. Observation
b. Having the pt drink a glass of milk
c. IV administration of glucagon
d. Sublingual administration of nifedipine
e. Endoscopy
e. Endoscopy

button batteries can cause perforation of the esophagus therefore they must be removed
What is the most common cause of small bowel obstruction?
a. Adhesions
b. Cancer
c. Foreign bodies
d. Crohn's disease
e. hernia
a. Adhesions
In women, the most common type of hernia is
a. Indirect inguinal
b. Direct inguinal
c. Incisional
d. Umbilical
e. Femoral
a. Indirect inguinal hernias

most common hernia in both men and women
Which one of the following statements regarding cholecystitis is true?
a. It is caused by impaction of a gallstone in the cystic duct
b. It is characterized by Charcot's triad
c. Antibiotic therapy should be started as early as possible
d. Jaundice is not a sign of cholecystitis
e. Eighty-five percent of gallstones are visible on plain films
a. It is caused by impaction of a gallstone in the cystic duct
Which one of the following statements about the ED management of hepatitis is true?
a. Pts with severely elevated liver enzymes should be admitted
b. Vit K should be administered if the pts PTT is elevated
c. Prochlorperazine is the antiemetic of choice
d. Most pts can be referred for outpatient evalution
e. Discharged pts should be instructed to avoid dairy products
d. Most pts can be referred for outpatient evalution
Abdominal aortic aneurysm is most commonly misdiagnosed as:
a. pancreatitis
b. nephrolithiasis
c. MI
d. lower back strain
e. diverticulitis
b. nephrolithiasis

nephrolithiasis causes renal colic. Any pt over 60 yo with symptoms suggestive of renal colic, but no history of renal colic should be suspected of having an AAA
A 10 yo boy presents with illness for the past 2 days. Which of the following symptoms is least likely to be a sign of appendicitis?
a. vomiting
b. anorexia
c. fever
d. abdominal pain
e. nausea
c. Fever

only 21% of pts with acute appendicitis have fever
A 58 yo man with rectal bleeding is brought to the ED by his wife. His vital signs suggest early hypovolemia. He passes a large amount of further bright red blood per rectum in the ED. He has no abdominal pain or tenderness on examination. Which of the following plans would be most optimal for this patient?
a. Nasogastric suction, placement of 2 large bore IVs and an operation
b. Administration of dobutamine and endoscopy
c. Placement of a foley catheter and further observation only
d. Administration of IV fluids, antibiotics, and packed RBCs
e. Administration of IV fluids, packed RBCs, and endoscopy
e. Administration of IV fluids, packed RBCs, and endoscopy

most likely a bleeding diverticula
What is the most common cause of intrinsic acute renal failure?
a. CHF
b. Urethral stricture
c. Ischemia
d. BPH
e. Dehydration
c. Ischemia

Acute tubular necrosis secondary to ischemia is the most common cause of intrinsic acute renal failure
What is the most common cause of urinary retention in men?
a. prostate cancer
b. Urethral stricture
c. Nephrolithiasis
d. neurogenic bladder
e. BPH
e. BPH
Which one of the following is the most likely value for the fractional excretion of sodium (FENa) in a patient with intrinsic acute renal failure?
a. > 2.0
b. 1.0-2.0
c. 0.5-1.0
d. < 1.0
a. > 2.0

intrinsic acute renal failure is characterized by the inability of the kidney to concentrate urine
Which antibiotic regimen is appropriate for the treatment of UTI in a patient in the 3rd trimester of pregnancy?
a. TMP-SMX 160/800 mg orgally, once daily for 7 days
b. Nitrofurantoin, 50-100 mg orally, 4x daily for 7-10 days
c. Cipro, 250 mg orally 2x daily for 3 days
d. Norfloxacin, 400 mg orally 2x daily for 3 days
e. Ofloxacin, 200 mg orally, 2x daily for 7 days
b. Nitrofurantoin, 50-100 mg orally, 4x daily for 7-10 days

Nitrofurantoin, Amoxicillin or Cefpodoxime is indicated for the tx of UTI in pregnancy
What is the test of choice for diagnosing testicular torsion?
a. Doppler ultrasound
b. Angiography
c. Radionuclide scanning with technetium 99m pertechnetate
d. MRI
e. Plain radiograph
c. Radionuclide scanning with technetium 99m pertechnetate

Doppler US is a quick test but has a high false negative rate
Which disease process may predispose patients to the formation of urinary stones?
a. Diabetes Mellitus
b. Irritable bowel syndrome
c. Osteoarthritis
d. Dermatomyositis
e. Inflammatory bowel disease
e. Inflammatory bowel disease
A 23 yo man presents to the ED complaining of scrotal pain. The pain began 4 days ago and has gradually worsened. The pt now feels hot and his scrotum is too tender to allow for an adequate examination. What is the most likely diagnosis?
a. Testicular torsion
b. Hydrocele
c. Epididymitis
d. Urethritis
e. Spermatocele
c. Epididymitis
What is the intial therapy in the management of a pt diagnosed with Fournier's gangrene?
a. IV antibiotics
b. Surgical debridement
c. Oral analgesics
d. Hyperbaric oxygen
e. Retrograde urethrography
a. IV antibiotics

Fournier's gangrene (aka idiopathic synergistic necrotizing fasciitis) is a rapidly spreading infection of teh scrotum that is tx with IV abx followed by surgical debridement
A 35 yo man presents to the ED b/c of a painless, ulcerated lesion on the shaft of his penis. His left inguinal lymph nodes are enlarged and painful; occasionally, a yellowish fluid drains from them. Which of the following is the most likely diagnosis?
a. Herpes
b. Genital warts
c. Syphilis
d. Molluscum contagiosum
e. Lymphogranuloma venereum (LGV)
e. Lymphogranuloma venereum (LGV)

LGV is a STD that manifests as a painless lesion accompanied by painful buboes
Which one of the following treatments is appropriate for a 25 yo man who was bitten by a dog while vacationing in Mexico City? (The dog was unprovoked, and the pts last tetanus shot was 2 years ago)
a. Wound cleansing and debridement, 20 IU/kg rabies immune globulin (RIG; half of the dose at the bite site, and half IM), and the 1st of 5 doses of human diploid cell vaccine (HDCV)
b. Wound cleansing and debridement, 10 IU/kg RIG IM, and the 1st of 5 doses of HDCV
c. Wound cleansing and debridement, 20 IU/kg RIG (half of the dose at the bite site and the other half IM), the 1st of 5 doses of HDCV, and 0.5 mL tetanus toxoid administered IM
d. Wound cleansing and debridement and the 1st of 5 doses of HDCV
a. Wound cleansing and debridement, 20 IU/kg rabies immune globulin (RIG; half of the dose at the bite site, and half IM), and the 1st of 5 doses of human diploid cell vaccine (HDCV)

assume a possible rabies exposure
A woman presents with a painless ulcer measuring 3 mm on her vulva, and has associated tender abdominal and inguinal lymphadenopathy. The woman also complains of mild lower abdominal discomfort and a clear vaginal discharge. What is the most likely diagnosis?
a. Condyloma lata
b. HSV infection
c. Lymphogranuloma venereum
d. Chancroid
e. Granuloma inguinale
c. Lymphogranuloma venereum

caused by Chlamydia trachomatis
An 8 yo boy in Oklahoma is brought to the ED over the 4th of July weekend b/c of fever, chills, malaise, arthralgias, and a headache. Physical exam reveals a maculopapular rash that is most prominent on his wrists and ankles. What is the most likely dx?
a. Meningitis
b. Lyme Disease
c. Rabies
d. Rocky Mountain spotted fever
e. Babesiosis
d. Rocky Mountain spotted fever

this is based on the rash on the wrists and ankles, where the boy lives, and the time of year
A 20 yo woman complains of progressive weakness in her lower extremities but denies having any other symptoms. Physical exam reveals an afebrile pt with absent deep tendon reflexes. What is the most likely diagnosis?
a. Rabies
b. tetanus
c. Relapsing fever
d. Tick paralysis
e. Lyme disease
d. Tick paralysis

presents clinically like Guillain-Barre syndrome. Removing the tick is curative
Which common parasite can cause rectal prolapse in children?
a. Ascaris lumbricoides
b. Enterobius vermicularis
c. Trichuris trichiura
d. Trichinella
e. Taenia solium
c. Trichuris trichiura

aka whipworm - acquired by children playing in contaminated soil
A pregnant woman who has been vacationing at Martha's Vineyard presents to the ED complaining of photophobia and a mild frontal headache. On further evaluation, she is found to have 3rd degree atrioventricular block. What is the appropriate treatment for this pt at this time?
a. Doxycycline (100 mg orally 2x daily for 14 days) and discharge
b. Amoxicillin (500 mg orally 3x daily for 14 days) and discharge
c. Chloramphenicol (50 mg/kg/day IV) and admission
d. Ceftriaxone (1 g IV every 12 hours), temporary pacemaker placement, and admission
e. Tetracycline (500 mg IV every 6 hours), temporary pacemaker placement, and admission
d. Ceftriaxone (1 g IV every 12 hours), temporary pacemaker placement, and admission

pt has stage II lyme Disease with neurologic symptoms of meningoencephalitis and cardiac symptoms with complete heart block
Which of the following drugs or fluids is not considered useful in the tx of septic shock?
a. Isoproterenol
b. Normal saline
c. Lactated Ringer's Solution
d. Dopamine
e. Norepinephrine
e. Norepinephrine

NorEpi causes intense peripheral vasoconstriction, myocardial hyperexcitatory activity, vital organ hypoperfusion and ischemic tissue necrosis thus its use in tx sepsis is RARE
A 69 yo woman complains fo weakness. The results of her labs are: Na - 122, K - 4.5, Cl - 110, Bicarb- 24. The serum Na level is noted to be chronic. What treatment is required?
a. Administer 4% saline at a rate of 100 ml/hr
b. Administer normal saline at a rate that does not exceed 0.5 mEq/L/ hr
c. Administer demeclocycline (600-1200 mg/day)
d. Administer furosemide
e. Admit the pt for observation
b. Administer normal saline at a rate that does not exceed 0.5 mEq/L/ hr

This is how you treat a stable pt with chronically low sodium. If the pt has acute, severe hyponatremia then administer 3% saline
Which of the following agents most rapidly counteracts the cardiac effects of severe hyperkalemia?
a. Calcium gluconate
b. Furosemide
c. Insulin-dextrose
d. Sodium Bicarbonate
e. Cation exchange resin
a. Calicum gluconate

Ca stabilizes cell membranes w/o altering K+ levels
An abnormally low serum level of which one of the following electrolytes results in prolongation of the QT interval?
a. Chloride
b. Sodium
c. Calcium
d. Potassium
e. Phosphate
c. Calcium

Hypocalcemia causes prolongation of the QT interval and may cause ventricular arrhythmias, but more commonly results in sinus bradycardia or complete heart block
What is the most common cause of high anion gap metabolic acidosis?
a. Ketoacidosis
b. Lactic acidosis
c. Persistent diarrhea
d. Pancreatic fistula
e. Anxiety
b. Lactic acidosis

results from decreased O2 delivery to the tissues
Pseudohyponatremia occurs secondary to which disorder?
a. Central nervous system disease
b. Addison's disease
c. Renal tubular acidosis
d. Hyperglycemia
e. Hypopituitarism
d. Hyperglycemia

Pseudohyponatremia is defined as a decrease in serum sodium concentration that is not accompanied by a decrease in plasma osmolality. It is seen with severe hyperglycemia, hyperproteinemia and hyperlipidemia
A 55 yo man with a history of lung cancer with metastasis to the pelvis presents with weakness, depression, confusion and abdominal pain. The serum Calcium level is noted to be 13 mg/dl. After initiating infusion of normal saline in the ER what is the next intervention?
a. Administration of mithramycin (15-25 mg/kg in 5% dextrose) IV over 3 hours
b. No additional treatment is required
c. Administration of calcitonin (2-4 IU/kg) IM every 12 hours
d. Administration of a cation exchange resin
e. IV infusion of insulin-glucose
a. Administration of mithramycin (15-25 mg/kg in 5% dextrose) IV over 3 hours

Pt requries tx for HyperCa b/c he is symptomatic and has a Ca over 12
A 20 yo pregnant woman receiving magnesium sulfate IV in the ED as tx for preterm labor begins to complain of nausea and weakness. On exam her deep tendon reflexes are depressed. What should be done for this patient?
a. Peritoneal dialysis should be performed
b. Furosemide should be administered
c. Magnesium sulfate infusion should be terminated
d. No tx is required
e. Calcitonin (2-4 IU/kg every 12 hours) should be administered
c. Magnesium sulfate infusion should be terminated
In acute respiratory acidosis, for each 10 mmHg increase in carbon dioxide tension (PCO2) the pH decreases by:
a. 0.08
b. 0.03
c. 0.02
d. 2
e. 5
a. 0.08

and the bicarb level increases by 1 mEq/L for every 10 mmHg increase in PCO2.

In chronic respiratory acidosis, the pH decreases by 0.03
In hypomagnesemia, clinical features mimic those of:
a. Hyponatremia
b. Hypercalcemia
c. Hyperkalemia
d. Hypocalcemia
e. Hypokalemia
d. Hypocalcemia

HypoMg and hypoCa are associated with hyperreflexia, tetany, seizures, Trousseau's sign and Chvostek's sign
Consider the following values: pH = 7.25, PCO2 = 40, PO2 = 92, HCO3 = 14. Which type of acid-base disturbance are they most consistent?
a. Respiratory alkalosis
b. Respiratory acidosis
c. Metabolic acidosis
d. Metabolic alkalosis
c. Metabolic acidosis
What is the mortality rate associated with Wernicke-Korsakoff syndrome?
a. 0-5%
b. 5-10%
c. 10-15%
d. 15-20%
e. 20-25%
d. 15-20%

Ophthalmoplegia and coma usually resolve but the memory deficit may persist
A 75 yo woman is brought to the ED. She has signs and symptoms of CHF, and a. fib is detected on EKG. What other symptom may also be observed?
a. Blepharoptosis
b. Exophthalmos
c. Stare
d. Lid lab
a. Blepharoptosis

Dx is most likely apathetic thyrotoxicosis, a rare form of thyroid storm in pts > 60 yo
What is the correct sequence of agents in the tx of thyroid storm?
a. Propranolol, propylthiouracil, iodine
b. Iodine, PTU, propranolol
c. PTU, iodine, propranolol
d. PTU, propranolol, iodine
c. PTU, iodine, propranolol

in pts in thyroid storm, synthesis of new hormone must 1st be prevented, followed by the retardation of thyroid hormone release. PTU is given 1st to inhibit thyroid hormone synthesis. Iodine is administered 1 hour later to prevent the release of thyroid hormone. Propranolol is used to block thyrotoxic symptoms and is given last.
How should insulin be administered to pts with nonketotic hyperosmolar coma?
a. Bolus
b. Bolus plus infusion
c. continuous infusion
d. No insulin should be administered
C. continuous infusion
A 50 yo man is brought to the ED following a drinking binge that has lasted for 3 days. Acute abdominal pain has caused the pt to temporarily stop consuming alcohol. The patient is found to have alcoholic ketoacidosis. What is the usual treatment?
a. Administer fluids and bicarb
b. Administer fluids, glucose and thiamine
c. Administer fluids, glucose, thiamine and bicarb
d. Administer fluids
b. Administer fluids, glucose and thiamine

Administration of saline with glucose is the mainstay of treatment for alcoholic ketoacidosis. Thiamine administration is indicated prior to giving glucose to prevent wernicke0korsakoff syndrome
What laboratory test assists in making the diagnosis of lactic acidosis?
a. Serum electrolyte panel
b. Serum lactate level
c. Serum ketone level
d. Serum glucose level
a. Serum electrolyte panel

Make the dx by finding an anion gap acidosis in the context of a process that can cause an accumulation of lactate. Elevated lactate levels are found in many conditions not associated with lactic acidosis
A 70 kg, 25 yo man is brought to the ED and a diagnosis of DKA is made. Administration of IV fluids is indicated. How much fluid should be given in the 1st 2 hours of therapy?
a. 500 ml
b. 1000 ml
c. 2000 ml
d. 10 ml/kg
e. 20 ml/kg
c. 2000 ml

Pts with DKA are significantly dehydrated and often have a fluid deficit of 5-10 L. Generally 2-3 L of fluid are administered during the 1st hour of therapy
Which one of the following is NOT commonly a precipitant of nonketotic hyperosmolar coma?
a. Hydrochlorothiazide
b. Cimetidine
c. Heat Stroke
d. Aspirin
d. Aspirin
The glasgow coma scale is used to evaluate a patient who has been brought to the ED. The pt opens his eyes only in response to painful stimuli and is making grunting sounds. The pt withdraws from painful stimuli. What is the glasgow coma scale score for this pt?
a. 3
b. 6
c. 8
d. 10
e. 12
c. 8

2 points for opening eyes to painful stimuli
2 points for incomprehensible sounds
4 points for exhibiting flexion withdrawal
A 70 yo man comes to the ED b/c he cannot see out of his right eye and has a right sided frontal headache.His ESR= 70 mm/hr. Immediate high dose steroid therapy is initiated to:
a. preserve visual acuity in the unaffected eye
b. prevent imminent cerebral infarction
c. reduce the ICp
d. prepare the pt for antibiotic administration
e. Reduce the inflammatory response caused by the tumor
a. preserve visual acuity in the unaffected eye

dx = temporal arteritis
A 60 yo man presents to the ED b/c of sudden onset of nausea, vomiting and vertigo. Exam reveals dysphasia and sagging of the soft palate on the right side. Cerebellar testing reveals right sided abnormalities with an abnormal heal to shin test and past pointing on the right. Motor and sensory examination findings are within normal limits. Where is the lesion most likely located?
a. Middle cerebral artery
b. Anterior cerebral artery
c. Posterior cerebral artery
d. Internal carotid arteria
e. Vertebrobasilar artery
e. Vertebrobasilar artery

Pt has lateral medullary (wallenberg) syndrome which results from occlusion of the posterior inferior cerebellar artery with the infarct localized to the brainstem and cerebellum
A pt with a known seizure disorder presents to the ED b/c he experienced a seizure the day before. The pt is taking phenytoin. While in the ED, the pt suffers a grand mal seizure. What actions should be taken at this time?
a. IV access should be established and benzodiazepines administered immediately
b. Pentobarbital anesthesia should be induced, followed by ET intubation
c. The pt should be protected from self harm
d. a loading dose of phenytoin should be administered IV
e. a CT scan should be performed immediately
c. The pt should be protected from self harm

this is the initial treatment
A pt with multiple sclerosis is asked to look to her right. She is able to move her right eye normally to the right but she cannot move her left eye past the midline. This pt may have a lesion of
a. nucleus of CN VI (left)
b. nucleus of CN VI (right)
c. nucleus of CN III (right)
d. nucleus of CN II (left)
e. medial longitudinal fasciculus (MLF; left)
e. medial longitudinal fasciculus (MLF; left)

Intranuclear ophthalmoplegia occurs when the MLF is ablated. the MLF connects teh nucleus of the contralateral CN VI with the nucleus of the ipsilateral CN III, resulting in conjugate gaze
Which of the following does NOT cause pinpoint pupils (miosis)?
a. pontine hemorrhage
b. Thorazine
c. Clonidine
d. Diphenhydramine
e. Fentanyl
d. Diphenhydramine

Diphenhydramine is an anticholinergic agent that induces mydriasis
A 65 yo male arrives by ambulence at the ED with severe headache after fainting and falling to the ground 30 minutes before. The CT scan is ordered adn it shows an acute finding. Which of the following diagnosis does NOT need urgent neurosurgical consultation and possible surgery?
a. cerebellar infarction
b. subdural hematoma
c. subarachnoid hemorrhage
d. embolic stroke
e. epidural hematoma
d. embolic stroke

embolic strokes can be tx medically
A 58 yo female presents with severe spinning of the room around her adn she is nauseated and vomiting. Which of the following is NOT typical of peripheral vertigo?
a. acute onset
b. nonfatigable nystagmus
c. absence of neurologic signs
d. Nausea and vomiting
e. Positive Nylen-barany maneuver
b. Nonfatigable nystagmus

peripheral vertigo is associated with N/V, hearing loss, absence of neurologic signs and fatigable nystagmus
a 23 yo secretary taking erythromycin for bronchitis ate cookies that someone had brought into the office. She later learned that the cookies contained walnuts. She had experienced prior urticarial reaction to walnuts and when she developed hives a few hours later she presented to the ED requesting medication for severe widespread pruritus. Which one of the following tx is most appropriate?
a. Ipecac
b. hydroxyzine
c. Topical diphenhydramine
d. prednisone
e. epinepherine
b. Hydroxyzine

1st line tx for acute urticaria is oral or IV antihistamine (hydroxyzine)
A pt reports a prior anaphylactic reaction to radiocontrast media. Which one of the following statements regarding this case is correct?
a. The risk for the 1st rxn was 5-15%, regardless of whether ionic or nonionic radiocontrast media was used
b. The chance of a second reaction would not be substantially reduced by pretreatment with corticosteroids and antihistamines
c. the previous rxn could not have been a vasovagal reaction
d. The recommended alternative for the future is to try to avoid studies involving radiocontrast media entirely
e. Nonionic radiocontrast media is the same price as ionic radiocontrast media.
d. The recommended alternative for the future is to try to avoid studies involving radiocontrast media entirely
A 38 yo pt presents with symptoms of L5 nerve root impairment that have been ongoing for 2 weeks. There is no history of trauma or fall. Examination and testing of this pt are likely to reveal which of the following?
a. Abnormal findings on plain radiographic films
b. Hypesthesia of the entire lower leg on the affected side
c. Sensory findings that may include diminished pain on the dorsum of the distal region of the foot
d. Motor findings that may include weakness on flexion of the great toe
e. Tendon reflexes expected to be diminished at the knee and ankle
c. Sensory findings that may include diminished pain on the dorsum of the distal region of the foot
A 40 yo male presents with knee pain. Joint effusion is aspirated, and synovial fluid shows a WBC count of 100,000, weakly positive birefringent rhomboid crystals and a gram stain negative for organisms. Which of the following is teh appropriate tx for this patient?
a. Reaspirate the joint to drain it completely and then inject decadron 10 mg IV
b. Inject glucocorticoid medication into the joint
c. prescribe prednisone and indomethacin and discharge the pt home
d. admit the pt to the hospital
e. Do not test the serum calcium level as it does not help diagnose
d. Admit the pt to the hospital

Weakly positive birefringent rhomboid shaped crystals on polarized microscopy suggest that Calcium pyrophosphate deposition disease or pseudogout is ongoing. A WBC count tho is suggestive of an infection. Best bet is to admit the pt, drain the joint, tx with NSAIDs, and consider Antibiotic coverage
Which of the paired clinical scenarios and test results is LEAST likely to have clinical significance?
a. Young woman with arthritis and rash - positive ANA test, high titer
b. Elderly woman with minimal symptoms - positive ANA, low titer
c. Pt with Raynaud's phenomenon and thickened skin - Positive ANA, low titer
d. 60 yo pt with new headache - elevated ESR
e. 42 yo pt with SLE and worsening arthritis - elevated ESR
b. Elderly women with minimal symptoms - psoitive ANA test, low titer

ANA tests are positive in more than 90% of pts with SLE. ANA may also be present but genearlly in lower titer in other diseases and in the asymptomatic elderly
Which treatment is LEAST conventional for manifestations of scleroderma?
a. General symptom control with glucocorticoids
b. Glucocorticoids for pericarditis
c. ACE inhibitor for HTN and renal disease
d. Glucocorticoids for myositis
e. NSAIDs for pericarditis
a. General symptom control with glucocorticoids
A 45 yo male presents with urticaria after ingesting ibuprofen for the 1st time this year. Which of the following mechanisms or pathways most likely explains his symptoms?
a. Arachidonic acid pathway
b. Complement pathway
c. Direct histamine release
d. Immunoglobulin E
e. Kinin system
a. Arachidonic acid pathway
A 22 yo male presents with a history of lower back pain. He admits to being an IV drug user. Your evaluation reveals an infection with inflammation or the SI joint. Which one of the following etiologies is most likely?
a. Neisseria gonococcus
b. Staphylococcus epidermidis
c. e. coli
d. Pseudomonas aeruginosa
e. staph aureus
d. Pseudomonas aeruginosa

IV drug users have a propensity to develop infection with p. aeruginosa
A 7 yo boy is brought to the ED for evaluation. His mother reports that the rash that is now visible on his trunk and extremities started on his face. She says that the rash was preceded by a high fever; a cough; red, watery eyes; and a runny nose. Physical exam reveals that the rash is maculopapular. What is the most likely diagnosis?
a. Roseola
b. Rubella
c. Rubeola
d. Varicella
e. Scarlet fever
c. Rubeola (measles)
What is the recommended treatment for a patient with Toxicodendron contact dermatitis and widespread inflammation?
a. Oral diphenhydramine
b. Colloidal oatmeal baths
c. calamine lotion
d. oral prednisone
e. topical corticosteroids
d. Oral prednisone

pts with severe toxicodendron contact dermatitis and widespread inflammation should get oral prednisone with a tapering dose
A herald patch precedes the generalized rash in which of the following disorders?
a. pityriasis rosea
b. Psoriasis
c. Tinea corporis
d. Drug eruption
e. Molluscum contagiosum
a. Pityriasis rosea

begins as a single, oval lesion 2-10 mm in diameter on the trunk of a proximal extremity. This is known as a herald patch
Which one of the following drugs could cause Stevens-Johnson syndrome?
a. Phenytoin
b. Carbamazepine
c. Cephalexin
d. Ciprofloxacin
e. Gentamicin
a. Phenytoin

SJS is caused by: phenytoin, phenobarbital, PCN, sulfonamides, phenothiazines, quinidine and salicylates
A 4 yo girl is brought to the ED b/c her mother is concerned about the development of golden brown, wheeping lesions on her arm. The child is afebrile and otherwise appears well. There is no lymphadenopathy present. Which one of the following statements about this disorder is correct?
a. Acute rheumatic fever is a common complication
b. Acute glomerulonephritis occurs in 20% of pts
c. Antibiotic therapy should cover staphylococcus and streptococcus
d. PCN is the oral antibiotic of choice
e. Topical antibiotic therapy with mupirocin is the most effective therapy
c. Antibiotic therapy should cover staphylococcus and streptococcus

Dx = Impetigo
A 38 yo woman with leukemia develops a painful rash involving her left eyebrow, her left eyelid, the left side of her forehead and her nose. The pt is undergoing chemotherapy for the leukemia. What is the most appropriate tx for this patient?
a. Corticosteroids
b. Antibiotic eyedrops
c. IV antibiotics
d. Oral acyclovir
e. IV acyclovir
e. IV acyclovir

Dx = Herpes Zoster (shingles)
A 50 yo man comes to the ED complaining of itching and a rash. Upon examination, multiple healing excoriations are noted. Two red, linear lesions are present on his feet. No cellulitis is present. The pt states that he had been in the ED 5 days previously with the same complaint, and he had been prescribed lindane lotion, which he used. The pt insists the lotion is not working and wants another prescription. What would be the next appropriate step?
a. Anotehr course of lindane should be prescribed
b. Permethrin should be prescribed
c. Erythromycin (500 mg orally every 8 hours for 10 days) should be prescribed
d. Diphenhydramine (25 mg orally every 6 hours) and calamine lotion should prescribed
e. Oral prednisone should be prescribed
d. Diphenhydramine (25 mg orally every 6 hours) and calamine lotion should prescribed

Dx = Scabies
Which one of the following statements concerning the treatment of tinea is true?
a. Tinea unguium of the toenails can be cured with a 6 to 9 month course of oral griseofulvin
b. Tinea capitis is treated with a tar shampoo
c. Ketoconazole, used as an alternative to griseofulvin in the tx of tinea corporis, can cause hepatotoxicity
d. Topical antifungal agents are poorly absorbed thru nails and should not be used
c. Ketoconazole, used as an alternative to griseofulvin in the tx of tinea corporis, can cause hepatotoxicity
Which one of the following clinical features is associated with roseola infantum?
a. febrile seizures
b. age less than 6 months
c. conjunctivitis
d. concurrent maculopapular rash and high fever
e. maculopapular rash preceded by a high fever
a. febrile seizures

Roseola infantum - seen in children b/w the ages of 6 months and 4 years. A high fever persists for 3-4 days preceds the development of a maculopapular rash on the trunk, buttocks, extremities and face. The rash begins as the fever subsides and fades in 1-2 days
What is the single most important step in the evaluation and treatment of chemical burns to the eye?
a. Antibiotic therapy to reduce the likelihood of a secondary infection
b. Copious irrigation with saline or water as soon as the pt presents to the ED
c. Slit lamp examination to assess the depth of the burn
d. Cycloplegia to dilate the pupil to increase flow of aqueous humor into the anterior chamber and decreased pain
e. Evaluation of visual acuity
b. Copious irrigation with saline or water as soon as the pt presents to the ED
Which one of teh following statements regarding epistaxis is true?
a. HTN is a risk factor for developing nose bleeds
b. Posterior bleeds occur in Kiesselbach's plexus
c. Anterior bleeds are easier than posterior bleeds to control but are less common
d. Pts with posterior bleeds require packing and admission to the hospital
e. A CBC, Pt and PTT should be sent for all pts with epistaxis
d. Pts with posterior bleeds require packing and admission to the hospital

Packing the posterior pharynx is associated with hypoxemia and respiratory arrest and rebleeding can occur. THus pts need to be admitted to the ICU. Posterior bleeds often involve branches of the carotid artery
Which one of the following statements regarding acute sinusitis is true?
a. Plain sinus radiographs provide more information than a CT scan of the sinuses
b. Ciprofloxacin is the antibiotic of choice for the tx of acute sinusitis
c. Maxillary sinusitis is often preceded by an apical abscess of an upper molar
d. The commonly cultured organism is Mycoplasma
e. The use of decongestants and steroids in the tx of acute sinusitis is strongly discouraged
c. Maxillary sinusitis is often preceded by an apical abscess of an upper molar

or by an upper respiratory tract infection
Which one of the following statements regarding the tx of dental injuries is true?
a. An avulsed tooth should be scrubbed to remove debris and replaced in teh socket as soon as possible
b. A fractured tooth that bleeds from teh center of the fracture should be cleaned with a dental pick and then filled and covered with dental wax
c. An avulsed tooth should be handled only by the enamel
d. An apical abscess is best treated with oral antibiotics and follow up in 2-3 days
e. A fractured tooth that has a yellow center and does not bleed can be treated at the next available dental appointment without any treatment in the ER
c. An avulsed tooth should be handled only by the enamel

The dental ligament attached to the root is delicate and must remain on the root in order for the tooth to survive
A 65 yo female complains of losing vision in her right eye suddenly. Which of the following approaches is NOT useful in the tx of central retinal artery occlusion (CRAO)?
a. Applying digital pressure to the globe
b. Anterior chamber paracentesis
c. Administering mannitol or glycerol
d. Administering acetazolamide
e. Administering calcium channel blockers
e. Administering Calcium channel blockers

CCB would reduce the BP causing the pressure head in the artery to decrease and lessening the chance that the embolus would be forced out of the artery
Uveitis resulting from which one of the following general causes should NOT be treated with steroids?
a. Autoimmune disease
b. Trauma
c. Idiopathic uveitis
d. Infection
d. Infection

Steroids can exacerbate the infection
Which one of the following statements regarding acute mastoiditis is true?
a. Altered mental status is common and usually d/t severe pain
b. In a pt with altered mental status, tx with IV antibiotics effective against the agents most often responsible for meningitis and brain abscesses should be started after obtaining a CT scan of the brain
c. A CT scan is not a reliable imaging study to diagnose mastoiditis
d. Radiographs of the mastoids are not reliable for ruling out mastoiditis
e. Complex mastoiditis with abscess formation is usually treated with myringotomy tube placement and oral antibiotics
b. In a pt with altered mental status, tx with IV antibiotics effective against the agents most often responsible for meningitis and brain abscesses should be started after obtaining a CT scan of the brain
What is a distinguishing feature of psychosis (as opposed to delirium)?
a. Agitation
b. Imparied cognition
c. Fever
d. Aural hallucinations
e. Acute onset
d. Aural Hallucinations

Psychosis is seen with primary psychiatric disorders as opposed to delirium which usually has an organic basis. Aural hallucinations are a hallmark of psychosis
What is the initial priority in dealing with physicially violent patients?
a. Establishing IV access
b. Restraining the pt to ensure safety
c. Obtaining a pulse oximetry reading
d. Obtaining a urine drug screen
e. Checking the glucose level
b. restraining the pt to ensure safety

This will help ensure everyones safety
Which one of the following electrolyte disturbances is mot likely in a bulimic pt?
a. Hypernatremia
b. Hypermagnesemia
c. Hypoglycemia
d. Hypercalcemia
e. Hypokalemia
e. Hypokalemia

Vomiting, laxative and diuretics all involve the loss of potassium rich fluids
Which one of the following medical conditions can easily be mistaken for panic disorder?
a. Gastroenteritis
b. supraventricular tachycardia
c. peptic ulcer
d. Grand mal seizure
e. cerebrovascular accident
b. supraventricular tachycardia

SVT can occur for brief periods of time, producing chest tightness and SOB. very similar to a panic attack
Where does suicide rank on the list of causes of death in young adults?
a. Last
b. First
c. Second
d. Thrid
e. Fourth
c. Second
Which one of the following pts is at the highest risk for completeing a suicide attempt?
a. A 20 yo stewardess who took 20 acetaminophen tablets and then called a friend
b. A 35 yo accountant who slashed his wrists in the hall at work
c. a 50 yo housewife who states that she wants to die but has no plan for achieving this goal
d. a 14 yo teenage who drinks 12 sodas and vomits on the carpet
e. A 65 yo unemployed single man who attempted to hang himself in the woods
e. A 65 yo unemployed single man who attempted to hang himself in the woods

Pertinent risk factors: Age, gender, marital status and employment status
It is necessary to sedate an agitated older pt without IV access. Which one of the following is the best regimen to use?
a. Haloperidol, 2-5 mg IM every 30 min to a maximum dose of 20 mg
b. Diazepam, 10 mg IM every 15 minutes to a maximum dose of 50 mg
c. Droperidol, 5 mg IM every 15 min to a maximum dose of 30 mg
d. Lorazepam, 1 mg orally every 45 min to a maximum dose of 5 mg
e. Trazodone, 100 mg orally every 30 min to a maximum dose of 1g
a. Haloperidol, 2-5 mg IM every 30 min to a maximum dose of 20 mg

Haloperidol is the option here that is most rapidly abosrbed IM
Which symptom is not usually experienced by pts with panic disorder?
a. Dysarthria
b. Sweating
c. Palpitations
d. Tingling in an extremity
e. Chest pain
a. Dysarthria
A 34 yo nulliparous woman comes to the ED complaining of abdominal cramping that has been present for the past 2 days and spotting that started 6 hours previously. The pts LMP was 12 weeks ago. The menses prior to that occured 4 weeks earlier and was normal. The pts vitals are stable and she is not orthostatic. There is suprapubic tenderness on abdominal examination. Pelvic exam reveals a small amount of cervical bleeding, but the os is closed. Tenderness is noted in the left adnexa. No mass is palpated. The pt states that she and her husband have attended an infertility clinic for 6 months and that 3 months prior to the onset of symptoms, she began taking a new medication. A urine BHCG is positive and the quantitative level is 1200 mIU (gestation of 5 weeks). A transvaginal sonogram reveals a dual sac in the uterus consistent with an IU pregnancy of 5 weeks gestation. What is the most likely dx?
a. Inevitable miscarriage
b. Molar pregnancy
c. Threatened miscarriage
d. Ectopic pregnancy
e. Torsed ovary
d. Ectopic pregnancy

Although the dual decidual sac sign which is suggestive of an intrauterine pregnancy, is prsent on US, a concomitant ectopic gestation cannot be definitely ruled out.
A 21 yo woman is brought to the ED b/c she has been experiencing abdominal pain for the past 12 hours that has been getting progressively worse. The date of her LMP was 8 weeks ago. The pt is found by paramedics to be pale and lethargic, with a systolic blood pressure of 70 mmHg. Upon arrival in the ED, her BP is unchanged. The pts abdomen is distended and rigid with percussion tenderness. What is the most likely diagnosis?
a. Threatened miscarriage
b. Intrauterine pregnancy
c. Ectopic pregnancy
d. Incomplete miscarriage
e. Torsed ovary
c. Ectopic pregnancy

Woman of childbearing age with HypoTN, abdominal distention and abd pain has a ruptured ectopic pregnancy until proven otherwise
A 17 yo primigravida, determined to be 16 wks gestation by dates and US, complains of an itchy, abnormal vaginal discharge. She denies pain, bleeding or dysuria. Exam reveals no fever and no abdominal findings, but a thin watery vaginal discharge is present and the cervix is erythematous and tender. The cervical os is closed. The uterus is grapefruit sized and nontender. There is no adnexal tenderness. Microscopic eval reveals a motile organism. What is the most likely Dx?
a. PID
b. Bacterial vaginosis
c. Foreign body
d. Trichomoniasis
e. Gonorrhea
d. Trichomoniasis
A 28 yo multiparous woman at 30 wks gestation by dates, arrives in the ED b/c she is concerned about the sudden onset of bright red vaginal bleeding and the passage of blood clots. She denies a history of trauma or pain. Her vital signs are stable on arrival. How should you proceed?
a. An immediate and thorough pelvic examination is indicated
b. Vaginal delivery should be induced
c. A c-section should be performed
d. The pt should be transferred to the OR to undergo examination
e. The pt should be referred to an obstetrician
d. The pt should be transferred to the OR to undergo examination

Dx is most likely placenta previa or placental abruption. Stabilization of the pt is the 1st priority
A 40 yo mother determined to be about 30 wks gestation by US performed 5 days ago arrives in the ED b/c she is alarmed by the sudden onset of an abnormal vaginal discharge. The pt has been healthy and has had no complications associated with the pregnancy. Her vital signs are stable. Exam reveals thin, clear vaginal fluid in the vault and a closed cervical os. A Nitrazine test is positive. Ferning is noticed microscopically. Fetal monitoring reveals a normal heart rate range with good beat-to-beat variability. What is the appropriate next step?
a. Phospholipid screening and admission for immediate delivery
b. Phospholipid screening, cervical cultures, and admission for fetal monitoring
c. Phospholipid screening, cervical cultures, and follow up with the onset of uterine contractions
d. Cervical cultures, antibiotics, and admission for fetal monitoring
e. Cervical cultures, fetal monitoring and discharge
b. Phospholipid screening, cervical cultures, and admission for fetal monitoring

Dx. Premature rupture of membranes
A 21 yo college coed comes to the ED complaining of fever, abd pain and an abnormal vaginal discharge that has been present for 2 days. She has lost her appetitie and is even refusing fluids b/c of the pain. Her LMP was 1 week ago and on time. She has never been pregnant or had any vaginal infections. She is sexually active with one partner and uses oral contraception. Exam reveals a fever of 39*C and lower abd tenderness but no peritoneal signs. Mucopurulent cervical discharge is noted from a closed os that is exquisitely tender. Marked bilateral adnexal tenderness is also appreciated. The pregnancy test is negative. Her leukocyte count is 15,000 /uL. Gram stain of the discharge reveals no bacteria. With is the most likely diagnosis and appropriate management?
a. Cervicitis; outpt oral abx and follow up
b. PID; inpt IV abx
c. PID; outpt oral abx and follow up
d. Salpingitis; IV abx in ED followed by an outpt oral regimen and follow up
e. Salpingitis; outpt oral abx and follow up
b. PID; inpt IV abx

b/c pt can't eat or drink she most likely will not be able to follow an oral antibiotic regimen
A 35 yo pregnant woman, determined to be at 40 wks gestation by dates and US, presents in active labor. She reports that a sudden "gush" of greenish, watery vaginal discharge occured several hours before arrival. Exam reveals fetal head presentation, bulging the perineum. What is the most appropriate action for the physician to take at this time?
a. Applying countertraction to the baby's head
b. Performing an episiotomy
c. Having the pt transferred to the Labor and Delivery suite
d. Suctioning the neonatal nasopharynx and pharynx
e. Asking the mother to push
a. Applying countertraction to the baby's head

Countertraction of the baby's head will reduce the risk of rapid head expulsion and damage to the perineal structures
A healthy, 30 yo pregnant woman determined to be at 31 wks gestation by dates and US, presents to the ED b/c she has been experiencing progressive headaches, blurred vision, and upper abd pain for 7 days. She also complains that her wedding ring no longer fits, and her shoes seem too tight. Exam reveals a BP of 140/110, retinal AV nicking, mild epigastric abd tenderness, and diffuse pitting edema. What is the clinical diganosis and tx?
a. Mild pre-eclampsia; begin therapy with methyldopa and arrange for outpt follow up
b. moderate pre-eclampsia; begin therapy with magnesium sulfate and labetalol or hydralazine and admit
c. Severe pre-eclampsia; begin therapy with magnesium sulfate and labetalol or hydralazine and admit
d. Pre-eclampsia; begin therapy with magnesium sulfate, hydralazine and furosemide and admit
e. Eclampsia; begin therapy with magnesium sulfate, hydralazine, and furosemide and admit
c. Severe pre-eclampsia; begin therapy with magnesium sulfate and labetalol or hydralazine and admit

The diastolic BP and symptoms suggest this is severe pre-eclampsia
A 35 yo monther of 2 arrives complaining of vision problems. The pt states that she has been well until 3 mornings ago, when she woke up seeing double. The pt denies blurred vision, loss of vision, headache and other focal neurologic findings. Her past medical hx is notable for lack of menstrual periods for the last 6 months. Her pregnancies were uncomplicated. "tunnel vision" (loss of peripheral vision) is noted on ocular examination. The pt visited her OB/GYN last week and had some blood tests drawn. the results of the tests were obtained and reveal that the pt has low prolactin, estrogen and gonadotropin levels. What is the most likely dx?
a. Genetic defect
b. Tumor
c. Asherman's syndrome
d. Polycystic ovarian disease
e. Pregnancy
b. tumor
An 18 yo college coed is brought to the ED on a sunday night by her roommate. The roommate says the pt has been acting "strange". The roommate had gone home for the weekend and had returned to find the pt crying on the floor. The pt is subdued, distracted, and intermittently tearful, and has said repeatedly, "i want to get tested. He touched me." what is the most appropriate course of action?
a. psychiatric consult in a quiet area
b. Complete pelvic exam with rape protocol
c. History and physical exam searching for evidence of trauma
d. Immediate notification of the police or CPS agency
e. Immediate notification of the pt's parents
c. History and physical exam searching for evidence of trauma

This is suggestive of Sexual assault. Injuries should be sought and adressed before procurement of evidence in accordance with the state rape protocol
Which one of the following signs and symptoms may be associated with foreign body aspiration syndrome in children?
a. Cyanosis
b. Wheezing
c. Absent breath sounds on the affected side
d. Decreased chest excursion on the side of the obstruction
e. All of the above
e. All of the above
Which one of the following statements regarding sudden infant death syndrome (SIDS) is correct?
a. SIDS is the most common cause of postnatal deaths in developed countries
b. Most cases of SIDS are seen almost exclusively in kids younger than 2-3 weeks of age
c. Autopsy findings are diagnostic of SIDS
d. Almost all children are normal prior to the event that results in SIDS
e. The incidence of SIDS is equal in full term infants and in preterm infants
a. SIDS is the most common cause of postnatal deaths in developed countries

Most cases are seen in infants older than 6 weeks
A previously healthy 15 yo boy develops a mild pneumonia with nonproductive cough and no evidence of effusion. He does not appear extremely ill, although some mild exertional dyspnea is noted. Pulse oximetry is normal. The most appropriate therapy is:
a. Amoxicillin sodium
b. Erythromycin base
c. TMP-SMX
d. cephalexin
e. Penicillin sodium
b. Erythromycin base

Most common cause of nonviral pneumonia in kids older than 5 yrs are mycoplasma and strep pneumo.
A 2 month old infant with moderately severe bronchopulmonary dysplasia who is on home O2 therapy is admitted to the hospital with fever, wheezing and moderate to severe respiratory distress. No obvious cyanosis is seen but nasal flaring and retractions are noted. A CXR shows hyperinflation and bilateral interstitial infiltrates with no alveolar process. The CBC is unremarkable, pulse Ox is 96% on supplemental O2 by nasal canula at 2 L/min. A nasal washing for RSV is positive. The most appropriate therapy is:
a. oral amoxicillin sodium
b. IV ampicillin
c. IV ceftriaxone
d. Oral erythromycin
e. ribavirin by aerosolization
e. ribavirin by aerosolization
A 3 yo crying child is brought to the ED. He is rubbing both ears and is somewhat inconsolable. Physical exam reveals an erythematous, bulging right tympanic membrane with no light reflex. Teh 2 most likely bacterial causes of this illness are:
a. Strep pyogenes and s. aureus
b. haemophilus influenza and s. aureus
c. H. influenza and strep pneumoniae
d. s. pneumo and s. aureus
e. moraxella caterrhalis and s. pyogenes
c. H. influenza and strep pneumoniae

Dx = acute otitis media
Most comon causes at this age are s. pneumo and h.influ
A 3 yo white boy presents to the ED with a sore throat, dysphagia of solids but not liquids, and tender anterior chain adenopathy that measures 3x3 cm. The most appropriate initial therapy in this kid is?
a. PCN sodium
b. cephalexin
c. amoxicillin
d. TMP-SMX
e. Ampicillin
a. PCN sodium

Dx - strep pharyngitis
A 15 mo girl is brought by her parents to the ED with a history of subjective fever, listlessness and a rash. Physical exam reveals an ill-appearing infant with a normal temperature, but an obvious petechial rash on the extremities and the trunk. The fontanelle is not palpated but positive Kernig and Brudzinski signs are present. The capillary refil is delayed at 4 seconds. The most appropriate next step in the management of this pt is:
a. Establishment of an IV with administration of a resuscitative bolus and administration of abx
b. CBC and blood culture
c. LP with CSF analysis
d. Gram stain and culture of an aspirate of petechiae
e. Securing the airway with an ET intubation
a. Establishment of an IV with administration of a resuscitative bolus and administration of abx
A 15 mo girl is brought by her parents to the ED with a history of subjective fever, listlessness and a rash. Physical exam reveals an ill-appearing infant with a normal temperature, but an obvious petechial rash on the extremities and the trunk. The fontanelle is not palpated but positive Kernig and Brudzinski signs are present. The capillary refil is delayed at 4 seconds. What is the most likely causative organism?
a. h. flu type b
b. strep pneumo
c. Neisseria meningitidis
d. group B B hemolytic strep
e. s. aureus
c. Neisseria meningitidis
Which one of the following statements is true regarding intussusception in children?
a. Most intussusceptions are ileocolic and may be confused with appendicitis, except that there are symptom free intervals
b. Bouts of abdominal pain are colicky to continuous, may last hours to days and are usually not associated with vomiting
c. Most kids are alert during the painful episode and appear playful and active
d. Masses representing the intussusception are not usually palpated
e. A barium enema may reveal a classic "coil spring" appearance but usually does not aid in the tx of the disease
a. Most intussusceptions are ileocolic and may be confused with appendicitis, except that there are symptom free intervals
Which one of the following statements regarding appendicitis is true?
a. Appendicitis occurs more frequently in children younger than 5 years of age
b. Appendicitis is often easy to diagnose; anorexia, pain, and vomiting usually point to the diagnosis
c. The incidence of perforation is high and exceeds the incidence in adult cases b/c of delay in presentation of symptoms and misdiagnosis
d. Laboratory studies are often helpful in the diagnosis and are useful in differentiating appendicitis from other causes of abnormal pain
e. Definitive diagnosis can usually be established by a period of observation; surgery is not necessary
c. The incidence of perforation is high and exceeds the incidence in adult cases b/c of delay in presentation of symptoms and misdiagnosis
What is used initially to replace fluid deficits in hypotensive pts with hemorrhage caused by hemophilia or von Willebrand's disease?
a. Whole blood
b. Factor VIII
c. Cryoprecipitate
d. Normal saline
e. Packed RBC
d. Normal saline

initial approach to these pts is the same as it is for all pts with acute hemorrhage
Which one of the following hematologic test results is most reliably present in a pt with mild von willebrand's disease?
a. Prolonged PT
b. Low von willebrand factor antigen
c. Prolonged bleeding time
d. Prolonged thrombin clotting time
e. Prolonged activated PTT
c. Prolonged bleeding time
In which one of the following cases is splenic immune activity likely related to RBC destruction?
a. A 22 yo man who does not show the expected response to packed RBC
b. A 32 yo woman with sickle cell anemia
c. A 23 yo woman with acute hemolysis after taking a sulfa drug to treat a UTI
d. A 50 yo man with syphilis and paroxysmal cold hemoglobinuria
e. A 21 yo man with thymoma and decreased levels of blood cells derived from erythroid precurors
a. A 22 yo man who does not show the expected response to packed RBC

pt probably received RH incompatible blood and has extravascular immune hemolysis in the spleen
Which one of the following represents a 1st line ED therapy for hypercalcemia associated with neoplastic disease?
a. Hemodialysis and mithramycin
b. IV phosphate and glucocorticoids
c. Peritoneal dialysis and oral phosphate
d. mithramycin and peritoneal dialysis
e. Saline and furosemide
e. Saline and furosemide
Which one of the following represents an appropriate indication for immediate transfusion of the named blood component?
a. A 58 yo man with a mechanical valve and a Hgb = 8 (pRBC)
b. A 34 yo woman with von Willebrand's disease (cryoprecipitate)
c. A 23 yo man with mucosal bleeding and thrombocytopenia caused by antiplatelet antibodies (platelets)
d. A 23 yo trauma pt with loss of 15% of his blood volume (pRBC)
e. An asymptomatic 45 yo pt with no known hemorrhage and a plt count of 9000/mm (plt)
e. An asymptomatic 45 yo pt with no known hemorrhage and a plt count of 9000/mm (plt)

A plt count < 10,000 is an absolute indication for transfusion to reduce the risk of intracerebral hemorrhage
Which one of the following bleeding patterns does NOT usually require factor replacement in hemophialiacs with factor levels lower than 30%?
a. Intra-articular hemorrhage
b. Intramuscular hemorrhage
c. Retroperitoneal blood accumulation
d. Traumatic lacerations
e. intracranial hemorrhage with stable neurologic examination
d. Traumatic lacerations

Lacerations are not characterized as a significant bleed in hemophiliacs
A pt is diagnosed with a bleeding disorder. The pt has an X-linked recessive disorder that causes factor VIII deficiency and so this pt has which of the following?
a. Hemophilia A
b. von Willebrand's Ds
c. DIC
d. Hemophilia B
e. hereditary spherocytosis
a. Hemophilia A

Factor VIII deficiency
An 18 yo male pt presents with diffuse mucosal hemorrhage, gross coagulation profile abnormalities and purpura fulminans, and is later found to have sepsis caused by Neisseria meningitides. Which of the following is the likely cause of the bleeding abnormalities?
a. Hereditary spherocytosis
b. von Willebrand's disease
c. DIC
d. Hemophilia D
e. Lupus erythematosus
c. DIC

Purpura fulminans and DIC are characteristic of severe infection with n. meningitidis
A pt presents to the ED with a near complete amputation of his left lower extremity. The initial intervention for this pt is?
a. Apply a tourniquet to stop the bleeding
b. Apply direct pressure to control the bleeding
c. Clamp off any obvious bleeding vessels
d. Assess the distal extremity for any pulses
e. ***** the pts airway and breathing
e. ***** the pts airway and breathing
A pt presents after a MVA at which he was found ambulatory at the scene. He is anxious, smells of alcohol and has a heart rate of 110 bpm, a BP of 110/90, a normal capillary refill and normal urinary output. This pt is assessed in which one of the following ways?
a. He is intoxicated and an early head CT should be planned to r/o head injury
b. He may be in class I hemorrhage (up to 15% of blood volume loss)
c. He may be in class II hemorrhage (15-30% of blood volume loss)
d. He may be in class III hemorrhage (30-45% of blood volume loss)
e. He may be in class IV hemorrhage (40% of blood volume loss)
c. He may be in class II hemorrhage (15-30% of blood volume loss)

Narrowed pulse pressure, normal capillary refill, anxiety or mild agitation and near normal urine output may be seen in pts with up to 800-1500 mL of blood loss (class II hemorrhage)
The estimated blood volume in an average adult is:
a. 7% of body weight (in kg)
b. 8% of body weight (in kg)
c. a greater percentage of body weight than that of a child
d. approximately 7L
a. 7% of body weight (in kg)
Zone I neck injuries are those in the region including which one of the following anatomic sites?
a. The thoracic inlet
b. The area b/w the clavicular head and the angle of the mandible
c. The area above the angle of the mandible
d. The area posterior to the sternocleidomastoid muscle
a. The thoracic inlet

Zone I = thoracic inlet and below the head of the clavicles
Zone II = above the clavicles to the angle of the mandible
Zone III = above the angle of the mandible
A pt presents to the ED after blunt head trauma with unknown loss of consciousness. His family reports that he was initially confused, became alert, but again appears less awake. He has bruising over the left temporal area. Which of the following is the most likely cause of his symptoms?
a. Epidural hematoma
b. Subarachnoid hemorrhage
c. Subdural hematoma
d. Diffuse axonal injury
e. Concussion
a. Epidural hematoma
A pt with a known history of alcohol abuse is brought to the ED after a fall in which he struck his head. A head CT reveals a crescent shaped hyperdense lesion. The correct diagnosis is?
a. epidural hematoma
b. Subarachnoid hemorrhage
c. subdural hematoma
d. diffuse axonal injury
e. Concussion
c. subdural hematoma

tearing of bridging veins
A 50 yo man presents to the ED after a high speed MVC. He has bruises on his head and is comatose. The head CT reveals no focal lesion. Which is the likely cause of his symptoms?
a. Epidural hematoma
b. Subarachnoid hemorrhage
c. Subdural hematoma
d. Diffuse axonal injury
e. Concussion
d. Diffuse axonal injury

D/t shearing or tearing of nerve fibers. Diffuse axonal injury is characterized by coma in the absence of a focal lesion and CT often appears normal
A 16 yo girli is involved in a 55 mph collision in her car and remembers flexing her neck hard. She sustains multiple injuries, and her neuro exam reveals motor paralysis, loss of pain and temperature sensation and preserved posterior column function (position sense, light touch, and vibration). Her diagnosis is:
a. Anterior cord syndrome
b. central cord syndrome
c. Brown sequard syndrome
d. Spinal shock
e. Neurogenic shock
a. Anterior cord syndrome

follows a cervical flexion injury that cuases compression of the anterior spinal cord
A young male gang member is stabbed in teh neck and suffers paralysis, loss of gross proprioception and vibration on the same side as the lesion and contralateral loss of pain and temperature sensation. His diagnosis is?
a. Anterior cord syndrome
b. Central cord syndrome
c. Brown Sequard syndrome
d. Spinal shock
e. Neurogenic shock
c. Brown Sequard syndrome
A 15 yo male is brought to the ED after tackling a large football player. He remembers being knocked backwards hard, and then had a burning sensation and weakness of his arms. His neurologic deficit of the upper extremities is more pronounced than that in the lower extremities with scattered sensory losses. His diagnosis is?
a. anterior cord syndrome
b. Central cord syndrome
c. Brown sequard syndrome
d. Spinal shock
e. Neurogenic shock
b. Central cord syndrome

follows hyperextension injury and is seen in pts with degenerative arthritis of the cervical spine
A 32 yo man comes to the ED complaining of wrist pain. He tripped over his son's bike in the driveway and fell forward, leading on his outstretched left hand. The pt is right handed. Physical exam reveals significant swelling of the left wrist. THe AP and lateral wrist radiographs show a thin, radiolucent, transverse line across the distal radius. There is no displacment, angulation or deformity. What is the most likely diagnosis?
a. Type I salter Harris fx
b. Second-degree wrist sprain
c. Monteggia fracture
d. nondisplaced radius fracture
e. colles' fracture
d. nondisplaced radius fracture
A 32 yo man comes to the ED complaining of wrist pain. He tripped over his son's bike in the driveway and fell forward, leading on his outstretched left hand. The pt is right handed. Physical exam reveals significant swelling of the left wrist. THe AP and lateral wrist radiographs show a thin, radiolucent, transverse line across the distal radius. There is no displacment, angulation or deformity. How should this pts injury be immobilized?
a. No immobilization is necessary
b. Compression dressing
c. Splint
d. Circumfrential cast
e. Surgical fixation
c. Splint
A 32 yo man comes to the ED complaining of wrist pain. He tripped over his son's bike in the driveway and fell forward, leading on his outstretched left hand. The pt is right handed. Physical exam reveals significant swelling of the left wrist. THe AP and lateral wrist radiographs show a thin, radiolucent, transverse line across the distal radius. There is no displacment, angulation or deformity. What should the pt be told at the time of discharge from the ED?
a. Your wrist will be back to normal in a few weeks
b. You should expect severe pain and increased numbness for the next few days
c. You can continue your usual activities but keep your wrist immobilized at night
d. Use ice as directed, elevated your hand and do not use your hand for a few weeks
d. Use ice as directed, elevated your hand and do not use your hand for a few weeks
What is the msot common hand infection?
a. Felon
b. Paronychia
c. Tenosynovitis
d. Cellulitis
e. Septic arthritis
b. Paronychia

infection of the nail base
The mother of a 4 yr old girl brings her daughter to the ED b/c she has noticed that the child has been avoiding using her right arm for the last 3 hours. The ER doctor diagnoses subluxation of the radial head. Which of the following statements is consistent with the diagnosis?
a. The mother states that earlier, she "yanked" her daughter out of the street while the 2 of them were walking to the bus stop.
b. The pt is crying and appears to be in distress, and the elbow is swollen and tender
c. The pt holds her elbow in full supination and extension
d. The radiograph revelas a supracondylar fracture of the elbow
e. Reduction of the injury requires sedation and traction
a. The mother states that earlier, she "yanked" her daughter out of the street while the 2 of them were walking to the bus stop.
Which one of the following statements regarding clavicle fractures is true?
a. They are rarely seen in kids
b. They usually occur in the lateral 1/3 of the clavicle
c. They are typically caused by a blow to the upper chest
d. The only immobilization required for nondisplaced clavicle fracture is a sling
e. Displaced clavicle fractures usually require surgical treatment and are often associated with many complications
d. The only immobilization required for nondisplaced clavicle fracture is a sling

most common fracture of childhood. They usually occur in the middle third and are caused by a blow to the shoulder. If the fx is displaced, only a figure eight brace is required
An 81 yo woman presents to the ED after a fall from a standing position. She complains of right knee pain and is unable to bear weight on her right leg. One physical exam her knee is nontender, her right hip is tender and her right leg is shortened, externally rotated and adducted. What is the most likely diagnosis?
a. Posteior hip dislocation
b. Knee dislocation
c. femoral shaft fracture
d. Intertrochanteric fracture
e. Slipped capital femoral epiphysis
d. Intertrochanteric fracture
Which one of the following statements regarding glenohumeral dislocation is true?
a. It is also called a "shoulder separation"
b. It usually cannot be detected by physical exam
c. The most common type is an anterior dislocation
d. Pts are not prone to recurrence of the dislocation
e. It is not associated with humeral fractures
c. The most common type is an anterior dislocation
Which type of fracture is rarely overlooked on a radiograph?
a. Tibila plateau fracture
b. Scaphoid (navicular) fracture
c. 5th metatarsal fracture
d. Nondisplaced suparcondylar fracture
e. Colles fracture
e. Colles fracture

angulated fracture with a characteristic dinner fork appearance
How does one examine the motor function of the hand?
a. The motor component of the radial nerve is tested by having the pt abduct his or her thumb against resistance
b. The motor component of the median nerve is tested by having the pt extend his or her wrist against resistance
c. The motor component of the ulnar nerve is tested by having the pt spread his or her fingers against resistance
d. The deep flexor tendon is assessed by having the pt flex each proximal interphalangeal (PIP) joint
e. The superficial flexor tendon is assessed by having the pt flex each distal interphalangeal (DIP) joint
c. The motor component of the ulnar nerve is tested by having the pt spread his or her fingers against resistance
The wound healing process that involves the accumulation of lymphocytes and granulocytes, and teh removal of debris from teh wound by macrophages is called?
a. Epithelialization
b. Immediate response
c. Inflammatory response
d. neovascularization
e. Collagen synthesis
c. Inflammatory response
The history of an injury can help determine tx required. Which one of the following statements matching history to treatment is correct?
a. Age of the wound is important b/c it can determine scar size after suturing
b. Mechanism of injury has little importance in wound care
c. The extent of injury is of minor importance b/c emergency physicians repair all wounds
d. The degree of contamination of the wound affects the risk of infection and the amount of wound care required
d. The degree of contamination of the wound affects the risk of infection and the amount of wound care required
A 67 yo diabetic man presents at the ED with a laceration to his hand from a kitchen knife. He is taking ASA for a heart condition and prednisone for COPD. He is a long-time smoker. Which one of the following statements regarding his history is correct?
a. As far as wound care is concerned, the pt is in a young age group
b. If his diabetes is under control, it is not a consideration in tx
c. His medications could adversely affect healing
d. B/c the wound is caused by a kitchen knife, it is a clean wound with little chance of infection
d. The pt's history of smoking is irrelevant
c. His medications could adversely affect healing

ASA may prevent plt aggregation and cause blood to accumulate in the wound after it is closed, increasing the risk of infection
A pt with a forehead wound requires anesthesia. Which one of the following statements is correct?
a. Topical local anesthetic may not be effective for this area
b. The pain of injection can be reduced by administering buffered lidocaine prior to infiltration
c. The pain of injection can be reduced by injecting the surrounding tissue with anesthetic quickly
d. An anesthetic agent containing epinephrine is not safe to use in this area
b. The pain of injection can be reduced by administering buffered lidocaine prior to infiltration
Which one of the following statements regarding irrigation of a wound is correct?
a. Antibiotics are more effective than dilution for preventing infection.
b. The goal of irrigating a wound is to make the wound sterile
c. Irrigation can be accomplished by slowing dripping saline solution into the wound
d. High pressure, high volume irrigation is the preferred method of irrigation
e. Only nonionic detergents should be used as irrigating solution
d. High pressure, high volume irrigation is the preferred method of irrigation

this is the most effective tx against infection
A 30 yo man falls out of the back of a pick-up truck, landing on his right knee. His only injury is a severe laceration to the right knee contaminated by several pieces of gravel and dirt. A proximally based flap, 5 cm on each side, is raised from the knee. The wound has irregular, devitalized edges, and there are sections of subcutaneous tissue with poor vascularization. Which of the following statements concerning this wound is true?
a. Foreign bodies can be left in the wound if they are small
b. A radiograph may be required b/c of the risk of foreign bodies in the knee joint and fractures to the underlying tissue
c. The sections of subcutanesous tissue with poor vascularization can be left alone b/c they are under the skin
d. The devitalized tissue on the edges of this wound does not require removal
e. This type of wound rarely needs to be treated in teh operating room
b. A radiograph may be required b/c of the risk of foreign bodies in the knee joint and fractures to the underlying tissue
A 25 yo woman is bitten on the hand by her cat. WHich one of the following statements regarding this injury is correct?
a. There is little chance of infection b/c cat bites have a low infection rate
b. Cat bites to the hand can be sutured closed acutely
c. Antibiotics are not required
d. B/c the bite wounds are small, they do not need to be opened and irrigated
e. The 1st f/u visit should take place in 12-24 hours
e. The 1st f/u visit should take place in 12-24 hours
A tennis player is swinging back for an overhand shot when he hits the back wall with his right index finger and crushes the fingertip b/w the wall and the racket handle. Which one of the following statements regarding treatement of this injury is correct?
a. If the tip is avulsed, it may be allowed to heal by secondary intention
b. If there is greater than a 25% subungual hematoma, the nail should be left in place
c. if the nail bed is lacerated, radiographs to screen for a distal phalanx fracture are contraindicated
d. The nail bed may be sutured with nylon sutures
a. If the tip is avulsed, it may be allowed to heal by secondary intention

Fingertip avulsions that do NOT involve the bone may be allowed to heal by secondary intention
Which one of the following statements regarding discharge instructions, medications and f/u care for emergency treatment of wounds is correct?
a. If the pt is not in pain on discharge from the ED he or she probably will not be in palin later
b. The wound should be cleaned daily with saline solution and fresh antibiotic ointment and dressing should be applied daily
c. Sutures to the face should be removed in 2 weeks
d. All pts with wounds, even minor wounds should be prescribed antibiotics
e. F/u instructions are required only for serious wounds
b. The wound should be cleaned daily with saline solution and fresh antibiotic ointment and dressing should be applied daily
Activated charcoal would be expected to avidly adsorb which one of the follwoing substances?
a. Lithium
b. Lead
c. Aspirin
d. Iron
e. Hydrocarbons
c. Aspirin

Activated charcoal does not adsorb metals, hydrocarbons or alcohols
The ocular damage that occurs as a result of methanol poisoning is secondary to the accumulation of which one of the following substances in the vitreous humor and optic nerve?
a. Oxalate
b. Glycolate
c. Lactate
d. Formate
e. Folate
d. Formate

Methanol is metabolized to formate which causes ocular toxicity
The use of flumazenil as an antidote in suspected benzodiazepine overdose may be indicated for which one of the follwoing pts?
a. A pt which significant CNS depression 2 hours after the ingestion of diazepam
b. A pt with a known seizure disorder who has ingested diazepam
c. A pt with CNS depression 2 hours after the ingestion of a cyclic antidepressant and diazepam
d. A pt with hallucinations, tachycardia and fever 2 hours after the ingestion of an antihistamine and diazepam
e. A pt with chronic diazepam therapy with potential physiologic tolerance and dependence who develops CNS depression 2 hours after the acute ingestion of an overdose of diazepam
a. A pt which significant CNS depression 2 hours after the ingestion of diazepam

Flumazenil may be of benefit in a pt with significant CNS depression who has ingested a single dose of a benzo. It is contraindicated in pts who have taken a benzo along with an epileptogenic agent
Deferoxamine mesylate is an antidote for which one of the follwoing toxins?
a. Acetaminophen
b. Benzodiazepines
c. Methanol
d. Iron
e. Lithium
d. Iron
A pt is suspected of having organophosphate poisoning. Which one of the following tests would be most specific for determining the extent of CNS cholinesterase inhibition?
a. Pralidoxime level
b. Plasma (pseudo) cholinesterase level
c. Acetylcholine level
d. Urine assay for organophosphate metabolites
e. Erythrocyte (true) cholinesterase level
e. Erythrocyte (true) cholinesterase level

This is found in brain tissue, nervous tissue and RBC thus it most specifically reflects cholinesterase activity in nervous tissue
What is the role of urinary alkalinization in the Tx of salicylate poisoning?
a. To "trap" ionized salicylate in the renal tubules, preventing reabsorption and enhancing excretion
b. To provide adequate diuresis
c. To enhance hydrogen ion excretion
d. To allow greater reabsorption of potassium from the renal tubules, in exchange for salicylate
e. To prevent the precipitation of salicylate induced myoglobin in the tubules
a. To "trap" ionized salicylate in the renal tubules, preventing reabsorption and enhancing excretion
What is the specific antidote for anticholinergic poisoning?
a. Flumazenil
b. Physostigmine
c. Atropine
d. Benzodiazepine
e. Antihistamine
b. Physostigmine
N-Acetylcysteine is an antidote for acetaminophen toxicity. Administration of N-acetylcysteine provides which one of the following detoxifying substances?
a. Glutathione
b. Glucuronide
c. Sulfate
d. Folate
e. Thiamine
a. Glutathione
What is the most common cause of death in pts with tricyclic antidepressant overdose?
a. Cardiac dysrhythmias
b. Status epilepticus
c. Rhabdomyolysis
d. Acute respiratory distress Syndrome
e. DIC
a. Cardiac dysrhythmias

All of these options can be caused by tricyclic antidepressants but the greatest toxicity is on the heart
Which one of the following statements regarding carbon monoxide poisoning is true?
a. Signs are easily recognizable on physical exam
b. Carbon monoxide poisoning can be detected by calculating the oxygen saturation, using the arterial oxygen tension as a basis for the determination
c. Cherry red skin is an early sign of carbon monoxide poisoning
d. Carbon monoxide poisoning affects the CNS and hear more than other organs
e. Carbon monoxide is treated by administrating an IV antidote
d. Carbon monoxide poisoning affects the CNS and hear more than other organs
Which one of the following medications is indicated in the tx of cyclic antidepressant overdose?
a. Procainamide
b. Sodium bicarbonate
c. Flumazenil
d. Quinidine
e. Physostigmine
b. Sodium bicarbonate

Helps treat dysrhythmias and hypotension
Which one of the follwoing statements regarding dystonic reactions is true?
a. Antiemetics can cause dystonic reactions
b. Dystonic reactions are treated with cholinergic agents
c. Dystonic reactions are characterized by muscular flaccidity
d. Dystonic reactions are life threatening
e. Dystonic reactions are most commonly caused by OTC medications
a. Antiemetics can cause dystonic reactions

Dystonic reactions are most commonly caused by agents that cause dopaminergic blockade in the CNS such as antiemetics and antipsychotics
Which one of the following statements regarding caustic ingestions is true?
a. Acids cause a liguefaction necrosis
b. Activated charcoal usually facilitates gastric decontamination
c. Steroids have been proven to be beneficial
d. Theorectically, alkali injuries penetrate deeper than acid injuries
e. Neutralization has been proven beneficial
d. Theorectically, alkali injuries penetrate deeper than acid injuries

Alkali injuries cause a liquefaction necrosis, which allows deeper penetration of the substance into the tissue
Which one of the following statements regarding hydrocarbon aspiration is true?
a. Children older than 5 years are most often affected
b. Coughing, gasping, and grunting are suggestive of hydrocarbon aspiration
c. The onset of symptoms is usually delayed for at least 12 hours
d. Hydrocarbons of low volatility and high viscosity are the most likely to cause aspiration
e. Hydrocarbon aspiration can be treated with an antidote
b. Coughing, gasping, and grunting are suggestive of hydrocarbon aspiration

Hydrocarbon aspiration is most common in kids < 5 yo who typically swallow low viscosity, high volatility hydrocarbons
A pt is brought to the ED late on a saturday night after "snorting" a large amount of cocaine at a party. The pt is very anxious. His vitals are Temp 40*C, HR = 130 RR = 22 BP 180/100. What should the emergency physician do next?
a. Administer activated charcoal
b. Administer IV propranolol
c. Administer oral propranolol
d. administer oral diazepam
e. Administer IV diazepam
e. Administer IV diazepam

Benzos are the mainstay of tx for cocaine intoxication
Hypothermia is defined as a core temperature below?
a. 32*C
b. 37*C
c. 35*C
d. 30*C
e. 28*C
c. 35*C

90*F
The last reflex to disappear in hypothermic pts is the same reflex that 1st appears with rewarming. THis reflex is the ?
a. Patellar (knee-jerk) reflex
b. Achilles reflex
c. diving reflex
d. biceps reflex
e. plantar reflex
a. Patellar (knee-jerk) reflex

The reflexes become hypoactive at a core temp of <32*C (90*F)
Which of the following accounts for the greatest precentage of body heat loss from a person who is dry, has a normal basal metabolic rate, and is exposed to windless, cool air?
a. Radiation
b. Conduction
c. Respiration
d. Convection
e. Evaporation
a. Radiation

accounts for 55-65% of heat loss in a cold climate for a pt who is dry, has normal basal metabolic rate and is exposed to windless, cool air
A confused 78 yo woman is found outside her home in January during subzero temperatures. Large blisters have developed on teh plantar and dorsal surfaces of her feet, and she has cold, mottled, dusky colored, sensationless toes. The diagnosis for this pt is frostbite. After rewarming her, the correct tx should include:?
a. rupturing all clear and hemorrhagic blisters and admitting her to the hospital
b. immediately amputating the toes that are w/o blood flow or sensation
c. Discharging her to home with instructions to keep the feet elevated and wrapped with a heating pad, and to return in 6 days
d. Admitting her to the hopsital for close observation and isolation from infectious agents whith possible eventual amputation
e. Debriding the bilsters, prescribing an analgesic, and discharging the pt to a nursing home immediately
d. Admitting her to the hopsital for close observation and isolation from infectious agents whith possible eventual amputation
The most efficient mechanism for the human body to dissipate heat in environmental temperatures at or above body temperature is:
a. convection
b. conduction
c. respiration
d. radiation
e. evaporation
e. evaporation

most efficient method to remove heat b/c of the vaporization energy lost
A 58 yo white man travels to Panama to perform research. While exposed to the hot humid climate, he develops minor swelling to the feet/ankles. What is the best tx for resolving heat edema?
a. elevating legs and rest
b. drinking more water and increasing salt intake
c. administering furosemide and other diuretics
d. donning tight stalkings/increasing salt intake
e. applying ice packs to feet
a. elevating legs and rest
What is the primary factor differentiating heat stroke from heat exhaustion?
a. heat stroke pts will always have a core body temp higher than 40*C (105*F)
b. heat exhaustion pts are able to sweat, but heat stroke pts stop sweating
c. heat stroke patients have an altered mental status
d. heat exhaustion pts usually do not have volume depletion but heat stroke pts most often are hypovolemic
e. an altered mental status is usually found in both heat stroke and heat exhaustion, but sweating is absent in heat stroke victims
c. heat stroke patients have an altered mental status

Heat stroke is defined as hyperpyrexia and neurologic symptoms or altered mental status
Which one of the following statements regarding classic vs exertional heat stroke is true?
a. exertional heat stroke victims are not at risk for rhabdomyolysis and acute renal failure
b. classic heat stroke victims are usually sweating when found
c. risk factors for exertional heat stroke include being elderly, being without a/c or fan, and polypharmacy
d. exertional heat stroke often occurs in previously healthy young people who have exercised or exerted themselves strenuously
e. people with psychiatric or chronic disease are rarely at risk for classic heat stroke
d. exertional heat stroke often occurs in previously healthy young people who have exercised or exerted themselves strenuously
Which one of the following medications is useful for treating neuroleptic malignant syndrome and malignant hyperthermia?
a. IV Calcium gluconate
b. Furosemide
c. Dantrolene
d. Diphenhydramine
e. CCB
c. Dantrolene

It lowers myoplasmic calcium levels and decreases muscular contraction in both these syndromes thus decreasing muscle rigidity and hyperthermia that occurs in these conditions
A mountain climber is ascending a very high mountain. He develops a bifrontal headache, anorexia, weakness and fatigue. That night he finds it difficult to sleep, and he develops a worsening fatigue and malaise. The likely Dx and potential tx options include:?
a. Acute mountain sickness; tx w/ acetazolamide, hydration, rest, possibly oxygen and if symptoms worsen, descent
b. Acute cerebral edema; tx with steroids, and sleep at higher altitudes each night until acclimated
c. AMS; tx with rest, increased fluids, breathing into a bag to increase CO2 levels and stimulate respiration, and ascent to acclimate quickly
d. Acute cerebral edema; no specific tx is necessary b/c the climber will get better as he climbs higher
e. acute high altitude pulmonary edema; tx with O2 and descent to a lower altitude
a. Acute mountain sickness; tx w/ acetazolamide, hydration, rest, possibly oxygen and if symptoms worsen, descent
What is the most lethal of all high altitude illness?
a. Acute mountain sickness
b. High altitude cerebral edema
c. High altitude pulmonary edema
d. Ultraviolet keratitis
c. High altitude pulmonary edema

most cases develop on days 2-4 after ascent above 14,500 ft
Most serious scuba diving injuries are a result of:
a. barotrauma
b. Decompression sickness
c. The bends
d. Dysbaric air embolism
e. Squeeze
d. Dysbaric air embolism

results from gas bubbles entering the systemic vasculature when pulmonary tissue rupture, causing strokes etc
A 4 yo gilr is found underwater in a swimming pool, unresponsive after being "missing" for several minutes. She has vital signs at the scene, but then dies en route to the hospital and is not resuscitated. The correct term for this girl's death is
a. Drowning
b. secondary drowning
c. Immersion syndrome
d. near-drowning
e. postimmersion syndrome
b. secondary drowning

Secondary drowning describes death that follows a brief period of recovery after the initial submersion event. Drowning is death by submersion in a liquid medium
Most venomous snakes in the US may be recognized by which one of the following features?
a. Round eyes
b. Absence of hollow fangs
c. Small "pit" indentation b/w each eye and nostril
d. Double row of caudal plates form the anal plate to one third of the way from the tail
e. Round, oval head
c. Small "pit" indentation b/w each eye and nostril

Most venomous snakes in the US are pit vipers and are recognized by a small "pit" indentation near each slit shaped eye
Lightning strikes to humans are associated with a mortality rate of:
a. 10%
b. 30%
c. 75%
d. 90%
e. 100%
b. 30%
Which of the following therapies falls within the EMT-B scope of practice?
a. Initiation of IV fluids
b. Administration of high-flow oxygen via a nonrebreather mask
c. Cardioversion of ventricular tachycardia with amanual defibrillator
d. Administration of IV dextrose for hypoglycemic coma
e. Administration of sublingual nitroglycerin for CP
b. Administration of high-flow oxygen via a nonrebreather mask

NON-invasive skills
Which one of the following is an example of retrospective medical direction?
a. Initial training of a paramedic
b. Developing a new protocol for the use of adenosine in SVT
c. Ensuring proper training of the paramedic in the use of this new drug
d. Discussing the use of adenosine with the paramedic when he or she calls for on-line medical command
e. Reviewing ambulance call reports to determine the trends of adenosine use and to be sure it is being used correctly
e. Reviewing ambulance call reports to determine the trends of adenosine use and to be sure it is being used correctly
Which one of the following is the most likely EMS activity for the general ED Doctor?
a. Performing an audit of a trauma registry for a large urban EMS
b. Providing direct (on-line) medical direction for an ALS unit in the field
c. Revising protocols to incorporate recent changes in the practice of prehospital medicine
d. Supervising an EMS fellowship program
e. Performing EMS research
b. Providing direct (on-line) medical direction for an ALS unit in the field
Which one of the following was included in the list of 15 essential EMS components that was developed in 1973?
a. Air medical services
b. Physician medical direction
c. Disaster planning
d. International EMS development
e. Recommended immunizations for EMS personnel
c. Disaster planning
A 1st responder can perform which one of the following medical interventions?
a. Emergency OB delivery
b. Delivery of oxygen via a nonrebreather mask
c. Cardiac monitoring
d. Extrication of the entrapped pt from a motor vehicle
e. Administration of IV fluids
a. Emergency OB delivery

1st responders are taught basic, lifesaving 1st aid, CPR and emergency delivery
Which one of the following missions would most likely be performed by an EMS helicopter?
a. Transport of a pt with a dissecting thoracic aneurysm from a rural community hospital to a tertiary center 35 miles away
b. Transport of a pt requiring a liver transplant to a specialty center 750 mi away
c. Medical coverage of a major league baseball game
d. Transport or a pt with CP from home to a local community hospital 10 miles away
e. Substitute for ground units during a snow storm
a. Transport of a pt with a dissecting thoracic aneurysm from a rural community hospital to a tertiary center 35 miles away
Which one of the following is closest to universal in EMS in the USA?
a. "911"
b. BLS ambulence service
c. ALS ambulance service
d. Public utility model EMS systems
e. Emergency physicians specializing in EMS
b. BLS ambulence service
Which one of the following incidents qualifies as a catastrophic disaster?
a. A subway derailment with 125 major injuries
b. A nerve gas attack at a professional baseball game with 35,000 injuries
c. An earthquake with significant damage to hospitals and numberous casualties
d. A fire aboard a cruise ship
e. A nursing strike at a community hospital
c. An earthquake with significant damage to hospitals and numberous casualties

Catastrophic disasters generally are considered to be those with destruction of infrastructures, making the provision of care difficult
Which one of the follwoing plans is the best hospital disaster plan?
a. One plan for internal disasters, one plan for external disasters
b. A physician plan, an administrative plan and a nursing plan
c. A plan that can expand as needed to fit the size of the event
d. A plan that emphasizes the steps needed to evacuate the hospital
e. A complex plan that accounts for every possible contingency on paper, leaving nothing to chance
c. A plan that can expand as needed to fit the size of the event
The pts in questions 1-4 all arrive at the same ED simultaneously, brought in by BLS ambulances from a MVA. It is 3 am and the ED physician has 2 nurses and a clerk to assist him. There is no physician backup in the hospital. The physician should triage these pts, matching each pt with the correct priority:
a. 1st priority
b. 2nd priority
c. 3rd priority
d. No tx

1. A 30 yo man with an open femur fx and facial contusions, a heart rate of 100 and a BP of 100/60 and evidence of moderate alcohol intoxication

2. A 45 yo woman with difficulty breathing, decreased breath sounds over the left chest, tracheal deviation to the right and a HR of 120

3. A 25 yo woman with no vital signs and evidence of closed head and abdominal injuries

4. A 4 yo boy, found by emergency workers in a child safety seat, crying loudly with contusions to both arms but no other apparent injuries and normal vital signs for his age
1 = b 2nd priority
2 = a 1st priority
3 = d No tx
4 = c 3rd priority
A 16 yo army private who sprained hsi ankle in a bball game wants to leave the ED to seek tx at the base clinic tomorrow. What should the ER doctor do?
a. Seek the pt's parents consent
b. Allow the pt to refuse tx
c. Have the pt sign out against medical advice
d. Insist on completeing all consolidated omnibus budget reconciliation act (COBRA) transfer paperwork
e. Seek the pts commanding officers consent
b. Allow the pt to refuse tx
A 15 yo runaway things she got "the clap" while working as a prostitute to support herself. she refuses blood tests. What hsould the ER doctor do?
a. Hold her in the ED until she reveals the names and whereabouts of her parents
b. Initiate tx w/ oral abx b/c she is a minor requesting tx for a STD
c. Notify the police that a missing person has been found
d. Consult CPS regarding a protective custody arrangement
e. Admit the pt to the pediatrics ward and consult the hospital attorney
d. Consult CPS regarding a protective custody arrangement
A 20 yo house painter fell 20 feet to the ground and was unconscious when the paramedics arrived. He is now conscious but unsteady on his feet. When questioned he is unable to relay what has happened to him or correctly identify the day of the week. Nevertheles, the pt insists that he wants to go home and "just rest" for his headache. The ER doc should?
a. Call the painter's employer to inform him that a work related injury has occurred
b. Call the painter's wife and ask her to consent for his treatment
c. Hold the pt against his will, continue to reason with him regarding the need for tx, and try to obtain a head CT scan
d. Call the police to place the pt on an inebriation hold
e. Let the pt sign out AMA if he is oriented and can walk unassisted
c. Hold the pt against his will, continue to reason with him regarding the need for tx, and try to obtain a head CT scan

Pt is NOT of normal mental capacity and easily may have a life threatening emergency
A 48 yo attorney is having crushing CP and is brought to the ED at Suburban hopsital. He tells the ER doc that he thinks the pains are stress-related and that he wants to go home b/c, ironically, he has to be in court tomorrow to represent a plaintiff in a malpractice case against Suburban. Which of the following would be the most prudent course of action on the part of the ER physician?
a. Offer to call the pts wife, his personal physicain, his clergy member or anyone else who might help persuade him to accept treatment
b. Allow the pt to sign out AMA b/c he is competent and an attourney
c. Allow the pts wife to drive him to another hospital
d. Notify the judge who will be heraing the malpractice case and tell him one of the attorneys in the case is ill
e. Admit the pt against his will as a threat to himself
a. Offer to call the pts wife, his personal physicain, his clergy member or anyone else who might help persuade him to accept treatment
A 34 yo science teacher has 4 bite marks on her hand. During questioning she admits that the injury occurred while feding her pet javelina. She tells the doctor that she brought the javelina across the border from Mexico on her last vacation and asks him not to tell anyone about it b/c they are illegal in this country. What should the physician do?
a. Call the police to report a crime
b. Treat the wounds like a cat bite and avoid documenting the type of pet in the chart
c. Respect the pts confidentiality but document that the pt was told she should call the local animal control agency to ascertain the legality of the pet
d. Call the local animal control agency to report the bite by an exotic animal and inquire as to the need for rabies vaccine
d. Call the local animal control agency to report the bite by an exotic animal and inquire as to the need for rabies vaccine
A 26 yo exterminator presents with several injuries including a possible hand fracture. The doctor notices blood on the back of the pts shirt. The pt is hesitant to allow the doctor to examine his back but eventually agrees. Upon examination the doctor discovers what appears to be a bullet wound. A CXR shows a small calliber bullet lodges above the pts right shoulder. The pt asks the physician not to report the injury saying that, " he can take care of the problem himself". What should the physician do?
a. Tx the pt and honor his statments as confidential b/w physician and pt
b. Call the police to report a pt with a bullet wound
c. Call the other hospital ED in the area to see if a similarily injured pt was brought in
d. Refuse tx until the pt reveals details of the assault
b. Call the police to report a pt with a bullet wound

In many states, injuries inflicted by weapons must be reported to police
A 57 yo pilot has a syncopal episode while urinating. Extensive workup reveals no cardiac or neurologic cause and the pt is D/c from the ED. Prior to D/C the pt asks the physician to bill him directly so he can pay cash for his tx. What should the physician do?
a. Tell the pt that according to hospital policy, pts insurance company must be billed
b. Direct the pt to the billing department to make whatever arrangements for payment he desires
c. allow the pt to pay cash but notify pts employer
d. allow pt to pay cash but notify the pts insurance company anyway
b. Direct the pt to the billing department to make whatever arrangements for payment he desires

Pts has a right to confidentiality
A 57 yo pilot has a syncopal episode while urinating. Extensive workup reveals no cardiac or neurologic cause and the pt is D/c from the ED. Prior to D/C the pt asks the physician to bill him directly so he can pay cash for his tx. 2 days later a man identifying himself as the company's physicain at Transwest airlines calls the physician who treated the pilot. He tells the doctor that he has been notified of the pts lacks of consciousness and he wants to verify it before suspending him temporarily. The pilot is scheduled to pilot flight 747 to Dallas tonight. The phsycian should?
a. Tell the caller no pt information is available over the phone but to question the employee directly
b. Direct the caller to medical records dept
c. tell the caller that no pt information is available over the phone and then contact the FAA after consulting with the hospital attorney
d. Tell the caller that pt information is not available over the phone but advise him to cancel the flight
e. Tell the call that with the pts written consent he can obtain a faxed copy of the pts chart
c. tell the caller that no pt information is available over the phone and then contact the FAA after consulting with the hospital attorney

It is the doctors job to warn the passengers on the flight
A 22 yo chronic schizophrenic is brought in to the ED by police for disturbing the peace. The police asked the doctor on duty to sedate the pt and hold her until morning. Can the physician tx this pt w/o her consent?
a. No- this pt has the right to refuse tx
b. No - although this pt may not be able to consent or refuse no life or limb threatening emergency exists
c. Yes - this pt is too impaired to consent
d. Yes - this pt has an acute psychiatric emergency
e. Yes - b/c the police have requested a tx it is legal to tx the pt
b. No - although this pt may not be able to consent or refuse no life or limb threatening emergency exists

Her outbursts do not pose a threat to herself or others
A 17 yo is brought in by police for weaving down the street while driving. No accident occurred and the pt pulled over when directed. According to the officer the pt failed a field sobriety test which involved walking a straight line. The officer plans to cite the teenage for driving under the influence of alcohol but not arrest him. The officer requests the physician obtain a blood sample from the pt. Can the physician carry out this request?
a. No- the pt can refuse or consent as long as he is not under arrest
b. No - the pts parents should be called for consent b/c he is a minor
c. No - the pt may refuse even if he is under arrest
d. Yes - the pt may not refuse a reasonable police request
e. Yes - otherwise the physician could be charged with obstructing justice
d. Yes - the pt may not refuse a reasonable police request
A 17 yo is brought in by police for weaving down the street while driving. No accident occurred and the pt pulled over when directed. According to the officer the pt failed a field sobriety test which involved walking a straight line. The officer plans to cite the teenage for driving under the influence of alcohol but not arrest him. The officer requests the physician obtain a blood sample from the pt. The father of this pt arrives in the ED and tells the doctor that his son has diabetes and that he had been due home from a date with his girlfriend. The girlfriend is contacted and she relates that they could not get seated at a restaurant in time so they went to a movie instead. She said her boyfriend seemed fine all night and that he dropped her off just minutes before he was stopped by police. The father demands the blood sample be relinquished to him b/c he never consented to tx for his son and threatens a lawsuit. How should the phsycian proceed?
a. Relinquish the blood sample
b. Tell the father he needs the blood sample to determine whether the pt is hypoglycemic
c. He should evaluate the pt for a medical emergency but retain a blood sample
d. Refer the father to the police
c. He should evaluate the pt for a medical emergency but retain a blood sample
A 73 yo woman with advanced alzheimers is found in a cold apartment by her landlord. The landlord says he thinks the pts grand-neice, who was her conservator, was caring for her and buying her food with the pts social security check. He does not know how to contact the neice. Can the pt be tx w/o her conservator's permission?
a. No- the conservator must consent to tx on her great-aunts behalf
b. No- only the pt can consent if the conservator is not available
c. yes - the landlord may consent for the pt if he agrees to pay the bill
d. Yes - b/c the pt is mentally impaired and suffering from hypothermia so implied consent applies
e. No- police consent must be obtained to tx the neglected elderly woman
d. Yes - b/c the pt is mentally impaired and suffering from hypothermia so implied consent applies

Tx the pt on the bases of implied consent b/c a potential medical emergency exists
The chief of surgery, the hopsital administrator and a nurse who witnessed an anesthesia resident injecting fentanyl in a back room arrive in the ED. They confront the resident and demand a comprehensive blood and urine drug screen as part of their investigation and disciplinary action. The resident seems dazed and has pinpoint pupils but refuses to cooperate. Can a spontaneously voided urine specimen be submitted for drug testing?
a. Yes - the resident has committeed a crime and time is of the essence to preserve evidence
b. Yes- physicians are obligated to report colleagues who may represent a hazard to pts and the specimen will provide necessary evidence
c. No - the anesthesiology resident may refuse to have any body fluid analyzed unless he consents
c. No - the anesthesiology resident may refuse to have any body fluid analyzed unless he consents

He can refuse to submit a urine sample although the pt may require tx for drug intoxication but no obvious emergency exists and no tx or diagnostic testing can be performed against his consent at this time
Urgent care clinic owned by the hospital is located across the street from the ED. A nurse at the urgent care clinic calls the ED requesting the ED provide a monitored bed for a 67 yo pt awaiting transfer to his HMO approved hospital. The pt presented to the clinic with jaw pain but his EKG reveals abnormalities. Why is transfering the pt to another hospital a bad coarse of action?
a. It is a COBRA violation to transfer the pt from the clinic to the ED
b. It is a COBRA violation to transfer the pt to another hospital w/o stabilizing
c. It is a COBRA violation to assertain that the pt belongs to an HMO before stabilizing tx is initiated
d. It is a COBRA violation to not accept the pt at a facility with a higher level of care
b. It is a COBRA violation to transfer the pt to another hospital w/o stabilizing

B/c the pt is on hospital property, there is an obligation to examine and stabilize
Just as a pregnant 22 yo woman with contractions 12 min apart is registering at the ED of hospital A, her insurance company calls after being notified by her husband. The ins co representative insists that the womans husband should drive the pt 2 miles to hospital A's sister hospital which has inpt OB services. Hospital A does not offer inpt OB services but shares an oncall list with the sister institution. How should this situation be handled?
a. Husband should be advised to drive his wife to the sister hospital
b. Husband and wife should be escorted to the sister hospital by a midwife with a delivery kit
c. An OB and neonatal transfer team should be requested to escort the pt to the sister hospital unless delivery is imminent
d. The oncall OB should be consulted to see if he will accept the pt in transfer and then the pt should be sent in an ambulence even if delivery is imminent
c. An OB and neonatal transfer team should be requested to escort the pt to the sister hospital unless delivery is imminent

Pt must be examined to determine if she is in labor. If she is in labor all evidence of the transfer law must be met including an written assertion by the ER doctor that states that transfer to another hospital outweights the benefits of staying at a facility with no OB services